1 - RCRMC Family Medicine Residency

Download Report

Transcript 1 - RCRMC Family Medicine Residency

Pain Managment
A 40-year-old male with chronic hepatitis C has
osteoarthritis in his knees that is beginning to limit
his activity. He asks you if he can take
acetaminophen for the pain.
Which of the following would be appropriate
advice? (Mark all that are true.)
Acetaminophen overdose is a leading cause of
fulminant liver failure in adults
Acetaminophen is excreted through the biliary
system
He can safely take up to 3 grams of acetaminophen
per day
NSAIDs are preferred over acetaminophen in
patients with chronic liver disease
Answer
• Acetaminophen overdose is a leading cause
of fulminant liver failure in adults
Acute acetaminophen overdose is a very common
problem in the United States, and when
unrecognized can lead to fulminant hepatic failure.
In healthy adult nondrinkers, acetaminophen is safe
taken chronically in doses up to 4 g/day. Adults who
drink excessively, those with chronic liver disease,
and those with malnutrition are at increased risk for
toxicity. Acetaminophen should be limited to 2 g/day
in these persons (B level recommendation). It
appears safe at this dosage, and is preferred over
NSAIDs in patients with chronic liver disease.
Acetaminophen is metabolized in the liver and
excreted by the kidneys.
A 40-year-old male has had low back pain for 2
years. He asks your advice concerning physical
therapy.
Which of the following would be appropriate
advice? (Mark all that are true.)
Prescribed exercise programs are the most
efficacious physical modality for chronic back pain
Transcutaneous Electrical Nerve Stimulation
(TENS) units produce modest benefits in pain
reduction
Regular massage therapy often produces lasting
benefits
While expensive, multidisciplinary rehabilitation
programs are clearly beneficial
Hydrotherapy is ineffective for chronic back pain
Answer
• Prescribed exercise programs are the most
efficacious physical modality for chronic
back pain
While expensive, multidisciplinary
rehabilitation programs are clearly
beneficial
Hydrotherapy is ineffective for chronic back
pain
It is difficult to analyze the evidence for the efficacy of physical therapy,
because improvement is affected by a patient's effort and motivation, as
well as the personal attention one gets from the physical therapist.
Randomized, controlled trials are difficult to perform and compare. On
this subject, systematic reviews and meta-analyses do not always agree.
A review of physical modalities for chronic back pain published in 2004
looked not only at efficacy, but also at the clinical significance of the
effect. Only exercise programs and multidisciplinary rehabilitation
programs (which can cost thousands of dollars) were shown to be
effective and clinically beneficial. Laser therapy, spinal manipulation, and
massage were shown to be mildly effective with little lasting clinical
benefit. Using the same criteria, TENS, magnets, ultrasound,
hydrotherapy, and traction were ineffective. There was too little evidence
to rank acupuncture, back schools, and lumbar supports.
Physical therapists not only administer modalities but also provide
functional assessments, patient evaluations, and patient education.
Therapists can specialize in areas such as neurologic rehabilitation, wound
management, or sports training.
Two weeks ago, a 30-year-old female with a history of lymphoma
underwent her sixth and last cycle of chemotherapy before radiologic
reevaluation. She has had chronic lymphoma-related back pain for the past
six months, adequately controlled by oxycodone/acetaminophen
(Percocet), 5 mg/325 mg, one to two tablets orally every 4 hours as
needed. During her last chemotherapy cycle she became neutropenic, and
was treated with filgrastim (Neupogen).
The patient comes to your office today for regular follow-up of
prednisone-induced hyperglycemia, and complains of severe bilateral
lower extremity pain. The pain started about 2 weeks ago and is mostly
over her shins. She tells you it is a constant, sharp pain, and that the
Percocet is not relieving her pain anymore.
What is the most likely cause of her new pain?
Pain secondary to increased cytokines from chemotherapy-related
tumorlysis
Neuropathic pain related to her prednisone-induced hyperglycemia
Osteoporosis-related pain from high-dose prednisone
Bone pain from increased bone marrow activity resulting from treatment
with filgrastim
Tolerance to Percocet secondary to chronic use
Answer
• Bone pain from increased bone marrow
activity resulting from treatment with
filgrastim
Filgrastim is used to treat neutropenia from chemotherapy or
bone marrow transplantation. It stimulates granulocyte and
macrophage proliferation and differentiation. One of its most
common side effects is bone pain (30% of patients) that
usually starts in the first 3 days of treatment.
Although tumor lysis can cause pain from increased
circulating cytokines, this pain is usually diffuse. Long-lived
hyperglycemia does cause neuropathic pain that is usually
described as burning, shooting, "pins and needles," and
"painful numbness." The onset is very rarely acute, however.
Osteoporosis does not cause pain.
Tolerance does not occur abruptly. Any pain escalation
should be thoroughly investigated before a diagnosis of
tolerance is made, especially when the pain has an acute
onset.
You decide to use a pain rating scale to help
guide the management of a 77-year-old female
with peripheral neuropathy who has daily
pain.
True statements regarding these scales include
which of the following? (Mark all that are
true.)
They are simple and easy to administer
They eliminate embellishment of pain
The emotional state of the patient can
influence the rating
There are valid scales which are useful in
children
Answer
• They are simple and easy to administer
• The emotional state of the patient can
influence the rating
• There are valid scales which are useful in
children
Pain rating scales are used by many physicians to measure intensity of
pain and monitor the effect of therapy. (C level recommendation, evidence
level II) They can be administered in a matter of minutes and are easy to
score. These scales can be adapted for any age group by substituting
words and/or pictures. Most children understand that 10 is greater than 2
and can use the simple 1-10 scale.
There are four scales in wide use: numeric rating scales, verbal rating
scales, visual analog scales, and pain drawings. Some clinicians use pain
diaries to further enhance the rating's accuracy and provide information on
function. One drawback is that the scales allow for embellishment and can
be skewed one direction or the other by someone with little pain
experience (Evidence level III). A teenager who is naive to pain might rate
the pain of a sore throat as a 10, or a patient seeking sympathy or
additional treatment might overrate the amount of pain he or she is having.
Emotions affect the pain experience and can also affect the rating. While
there is some controversy about how accurate these scales are, most
clinicians have learned to use them for assessment and management of
pain.
40-year-old nurse presents with neck pain. He
has no history of specific injury. His
symptoms are intermittent and seem worse
when turning his head to the left. The pain
radiates into the left thumb and index finger.
At times it can be severe.
Which one of the following is true regarding
this problem?
The irritated nerve root is most likely
C5 Diagnostic imaging would be helpful
Flexion of the neck is likely to worsen the
pain
The most common cause is osteophytes
Answer
• Diagnostic imaging would be helpful
Cervical radiculitis is quite common and results from
irritation of the cervical nerve as it leaves the spinal cord. The
location of the symptoms may help to identify the irritated
nerve. A C5 root irritation will cause pain in the shoulder but
without radiation into the arm. The pain is generally made
worse by extension, a maneuver that decreases the space for
the nerve root. Flexion may actually help the pain. The pain is
generally also made worse by turning toward the side of the
compression. Reflexes decrease as the duration of the
compression lengthens.
The most common cause of cervical nerve root compression
is either an acute disc herniation or a degenerative disc.
Osteophytes may cause no impingement. Diagnostic imaging
is helpful and should start with plain films of the cervical
spine with oblique views (C level recommendation). An MRI
is the most definitive imaging test, but should be used to
confirm the diagnosis since there is a fair amount of
asymptomatic cervical pathology (Evidence level II).
Cervical Radiculopathy
• C5 impairment can send pain over the top of the shoulder
in the first fourth of the arm which is also where
numbness occurs, when present. When there is weakness,
it involves the ability to elevate the arm
sideways to the level of the shoulder or above. There are
no good (rubber-hammer-type) reflexes the
doctor can use to test this root.
C6 impairment can send pain as far as the thumb which is
also where numbness occurs, when present.
When there is weakness, it involves the ability to bend the
elbow. The doctor can additionally test for C6
impairment with the biceps-reflex which involves striking
a tendon in the crook of the elbow.
Cervical Radiculopathy
• C7 impairment can send pain as far as the middle fingers
which is also where numbness occurs, when
present. When there is weakness, it involves the ability to
straighten the elbow. The doctor can
additionally test for C7 impairment with the triceps-reflex
which involves striking a tendon on the back of
the elbow.
C8 impairment can send pain as far as the little finger
which is also where numbness occurs, when
present. When there is weakness, it involves certain handmovements, including the ability to join the tips
of the thumb and the little finger and also to spread the
fingers sideways. There are no good reflexes the
doctor can use to test this root.
Which one of the following
agents provides the greatest
analgesic effect when compared
milligram to milligram?
Oxycodon(OxyContin)
Morphine(MSContin)
Codeine
Hydromorphone(Dilaudid)
Hydrocodone
Answer
• Hydromorphone (Dilaudid)
True statements regarding the management of
vertebral compression fractures in the elderly include
which of the following? (Mark all that are true.)
These fractures are normally unstable, making
surgical treatment ideal
If conservative treatment is selected, a minimum of 4
weeks of bed rest is required
In the elderly, NSAIDs are safer than opioids
Calcitonin-salmon (Miacalcin) nasal spray can be
used for treatment of pain
Percutaneous vertebroplasty can be helpful in
managing the pain when conservative treatment fails
After the fracture heals, returning to a normal
exercise program may still be dangerous
Answer
• Calcitonin-salmon (Miacalcin) nasal spray
can be used for treatment of pain
Percutaneous vertebroplasty can be helpful
in managing the pain when conservative
treatment fails
Compression fractures of the vertebral body are common, especially in older adults. Vertebral compression fractures
usually are caused by osteoporosis, and range from mild to severe. More severe fractures can cause significant pain, leading
to the inability to perform activities of daily living, and life-threatening decline in the elderly patient who already has
decreased reserves. While the diagnosis can be suspected from the history and physical examination, plain radiographs are
often helpful for determining the diagnosis and prognosis. Occasionally, it may also be helpful to obtain CT or MRI.
The physician must first determine if the fracture is stable or unstable. A stable fracture will not be displaced by
physiologic forces or movement. Fortunately, compression fractures are normally stable as a result of their impacted nature.
Traditional treatment is non-operative and conservative. Patients are treated with a short period (no more than a few days)
of bed rest. Prolonged inactivity should be avoided, especially in elderly patients. Oral or parenteral analgesics may be
administered for pain control, with careful observation of bowel motility. If bowel sounds and flatus are not present, the
patient may require evaluation and treatment for ileus. Calcitonin-salmon nasal spray can be used for treatment of pain.
Muscle relaxants, external back braces, and physical therapy modalities also may help (Evidence level B). NSAIDs have
been shown to significantly increase gastrointestinal bleeding in the elderly and must be used with caution (Evidence level
A, randomized controlled trial).
Patients who do not respond to conservative treatment or who continue to have severe pain may be candidates for
percutaneous vertebroplasty. This involves injecting acrylic cement into the collapsed vertebra to stabilize and strengthen
the fracture and vertebral body. This procedure does not, however, restore the shape or height of the compressed vertebra.
Kyphoplasty, where cement is injected into a cavity created by a high-pressure balloon, is being evaluated for use and may
be successful in restoring height to the collapsed vertebra.
Most patients can make a full recovery or at least significant improvements within 6-12 weeks, and can return to a normal
exercise program after the fracture has fully healed. A well-balanced diet, regular exercise program, calcium and vitamin D
supplements, smoking cessation, and medications to treat osteoporosis (such as bisphosphonates) may help prevent
additional compression fractures. Age should never preclude treatment.
There is now good evidence that diagnosing and treating osteoporosis does indeed reduce the incidence of compression
fractures of the spine (Evidence level A). Regular activity and muscle strengthening exercises have been shown to decrease
vertebral fractures and back pain. Measures to prevent falls must be initiated by patients and their caregivers.
Family physicians can help patients prevent compression fractures by diagnosing and treating predisposing factors,
identifying high-risk patients, and educating patients and the public about measures to prevent falls.
You are seeing a colleague's patient for a follow-up
visit. The patient has been taking opioids for 12
months for chronic low back pain. After reviewing
his chart you notice numerous phone messages from
the patient asking for early refills, as he has been
using his opioids more frequently than prescribed.
During the encounter, the patient admits to
borrowing pain medications from his wife.
In the absence of other aberrant behavior, this is
highly indicative of
dependence
addiction
pseudoaddiction
drug trafficking
Answer
• pseudoaddiction
The suspicion of opioid addiction in chronic pain sufferers is often triggered by the occurrence of what have been called
aberrant drug-related behaviors. Ambiguities inherent in this approach affect patient care adversely. Rather than
consistently signifying abuse or addiction, these behaviors are often motivated by undertreated pain.
The term pseudoaddiction was coined in 1989 to describe chronic pain victims mistakenly diagnosed as suffering from
opioid addiction when undertreated pain led to certain drug-related behaviors. Simply stated, pseudoaddiction is a
misdiagnosis that results from undertreatment of chronic pain. Patients are frequently harmed by the misdiagnosis of
addiction and these behaviors should prompt an aggressive search for undertreatment of pain. Unfortunately, this usually
does not happen. Instead, when a patient displays certain behaviors, he or she is typically threatened with termination of
treatment, rather than questioned about its effectiveness.
Undertreatment of chronic pain should be considered first on the list of differential diagnoses when considering the cause of
worrisome drug-related behaviors. Some of these behaviors include
borrowing another patient's drugs
obtaining prescription drugs from nonmedical sources
unsanctioned dosage escalations
aggressive complaining about the need for higher doses
drug hoarding during periods of reduced symptoms
requesting specific drugs
acquisition of similar drugs from medical resources
(Evidence level III)
The diagnosis of opioid addiction should be based on observation of deteriorating function, which can be directly attributed
to opioid abuse, rather than inferred from an anecdotal set of behavioral criteria derived from medical folklore. Behaviors
suggestive of opioid addiction include injection of substances prescribed for oral use, concurrent use of related illegal
drugs, and selling prescription drugs (Evidence level III).
When patients are obtaining opioids from more than one medical source, the primary physician must reevaluate the pain
syndrome. Factors to consider include whether the patient is undermedicated, whether the syndrome is misdiagnosed, and
whether the patient is abusing or diverting drugs.
Physicians must work with chronic pain patients to adequately evaluate and treat their pain. In turn, physicians must expect
patients to use only one source to obtain opioid prescriptions. A written opioid use agreement is recommended.
Mind-body therapy (MBT), such as relaxation, (cognitive)
behavioral therapies, meditation, imagery, biofeedback, and
hypnosis, is used for several common clinical conditions.
There is good evidence to support which of the following
statements about MBT? (Mark all that are true.)
MBT is more effective for decreasing pain intensity than for
improving functional status associated with low back pain
MBT has NO significant effect in the symptomatic treatment
of arthritis
Stress management training can be as effective as tricyclic
antidepressants in the management of chronic tension-type
headache
The combination of relaxation training and thermal
biofeedback is the preferred behavioral treatment for
recurrent migraine disorder
Answer
• MBT is more effective for decreasing pain
intensity than for improving functional
status associated with low back pain
Stress management training can be as
effective as tricyclic antidepressants in the
management of chronic tension-type
headache
The combination of relaxation training and
thermal biofeedback is the preferred
behavioral treatment for recurrent migraine
disorder
Multimodal mind-body therapy (MBT) treatments typically include some combination of
relaxation, biofeedback therapy, cognitive strategies (e.g., for coping with pain), and
education. Narrative reviews suggest that the Arthritis Self-Management Program (ASMP)
might be a particularly effective adjunct in the management of arthritis (Evidence level
III). This community-based program consists of education, cognitive restructuring,
relaxation, and physical activity to reduce pain and distress and facilitate problem solving.
Using this program, reductions in pain were maintained 4 years after the intervention, and
physician visits were reduced by 40% (Evidence level II).
A review of the efficacy of MBTs in chronic low back pain concluded that there was
strong evidence (defined as generally consistent findings in multiple high-quality
randomized, controlled trials) that MBTs, when compared with wait-list controls or usual
medical care, have a moderate positive effect on pain intensity and only small effects on
functional status and behavioral outcomes (Evidence level I, Cochrane review).
A review of the efficacy of relaxation and biofeedback in recurrent migraine headache
showed a 43% reduction in headache activity in the average patient compared with a 14%
reduction with placebo medication and no reduction in unmedicated subjects (Evidence
level II). A more recent narrative review concluded that a combination of relaxation
training and thermal biofeedback is the preferred behavioral treatment for recurrent
migraine disorder (C level recommendation). Recent evidence indicates that stress
management training is as effective as tricyclic antidepressants in the management of
chronic tension-type headache, suggesting that combining these two therapeutic
approaches might be more effective than using either one alone (Evidence level I).
True statements regarding dysmenorrhea
include which of the following? (Mark
all that are true.)
Leiomyomata can cause secondary
dysmenorrhea
Oral contraceptives will not help primary
dysmenorrhea
NSAIDs can be used on an intermittent
basis to help with dysmenorrhea
Prostaglandins play a principal role in
dysmenorrhea
Answer
• Leiomyomata can cause secondary
dysmenorrhea
NSAIDs can be used on an intermittent
basis to help with dysmenorrhea
Prostaglandins play a principal role in
dysmenorrhea
Dysmenorrhea is pain that occurs during the menses and is
crampy in nature. It is commonly classified as either primary
or secondary. Primary dysmenorrhea is a condition unto itself
that is not a symptom of another disorder. Secondary
dysmenorrhea can be caused by leiomyomata or by other
pelvic pathology. Prostaglandin release is the understood
pathophysiology for primary dysmenorrhea. Oral
contraceptives provide relief for primary dysmenorrhea by
suppressing ovulation and thereby reducing the release of
prostaglandins (Evidence level II). NSAIDs that inhibit
prostaglandin synthetase provide relief in most patients and
are usually initiated for 2-5 days, just before and during the
menses (Evidence level I). In some recalcitrant cases the
NSAIDs could be used continuously, with proper attention to
the risks of chronic NSAID use.
A 34-year-old female presents with intermittent facial pain.
The pain occurs in brief episodes and always on the left side
of her face. She reports that the pain is like an electric shock.
The episodes may be evoked by smoking, talking, or washing
her face. There are times, however, where there is no inciting
stimulus. Between episodes she is pain free and there are no
sensation deficits on her face.
True statements regarding this problem include which of the
following? (Mark all that are true.)
Facial sensory loss associated with facial pain should prompt
cerebral imaging
This may be the first manifestation of multiple sclerosis
A large proportion of cases are caused by compression of the
nerve by a blood vessel
Carbamazepine (Tegretol) is first-line medical management
Patients not responding promptly to pharmacotherapy should
be offered referral for interventional therapy
Answer
• Facial sensory loss associated with facial
pain should prompt cerebral imaging
This may be the first manifestation of
multiple sclerosis
A large proportion of cases are caused by
compression of the nerve by a blood vessel
Carbamazepine (Tegretol) is first-line
medical management
Patients not responding promptly to
pharmacotherapy should be offered referral
for interventional therapy
Trigeminal neuralgia (TGN) is a painful condition which affects one side of the face. It is
characterized by brief shock-like pain limited to the distribution of one or more divisions of
the trigeminal nerve. The pain may be stimulated by such actions as washing, shaving,
smoking, talking, or brushing the teeth, but may also occur spontaneously. It begins and
ends abruptly, and may remit for varying periods.
Loss of facial sensation or any suspected involvement of a cranial nerve should prompt
appropriate cerebral imaging (C level recommendation). In the last three decades, evidence
has been mounting that in a large proportion of cases, compression of the trigeminal nerve
root at or near the dorsal root entry zone by a blood vessel is a major causative or
contributing factor (Evidence level III). Of the known etiologic factors, the association of
multiple sclerosis (MS) with TGN is well established. MS is seen in 2%-3% of patients with
TGN. Conversely, TGN is diagnosed in 1%-5% of patients with MS. In a small proportion
of patients with MS, TGN is the first manifestation of the disease.
Pharmacotherapy remains the mainstay of treatment of TGN. Unfortunately, only a few
randomized, controlled trials have been conducted. Carbamazepine (Evidence level I),
oxcarbazepine (Evidence level II), phenytoin (Evidence level III), lamotrigine (Evidence
level II), and baclofen are commonly used to treat TGN. Patients with TGN are often willing
to consider surgery as a first-line treatment in anticipation of a permanent cure. Numerous
interventional procedures (e.g., cryotherapy, alcohol blocks, radiofrequency lesions) and
operations (e.g., microvascular decompression) are available to treat TGN. Each is
associated with complications and recurrences. Patients should be provided a realistic view
and balanced information regarding treatment choices.
A 52-year-old male is admitted to the hospital with abdominal
pain and dehydration, and is diagnosed with inoperable
pancreatic cancer. He chooses to return home with hospice
care. While in the hospital, he has been using patientcontrolled analgesia (PCA). His PCA is set to deliver 1 mg of
intravenous morphine on demand, with a 10-minute lockout.
There is no basal rate. He reports his pain is well controlled.
Over the past 3 days, his morphine use has been 28 mg/day,
32 mg/day, and 29 mg/day.
You wish to send the patient home on sustained-action
opioids to help control his pain. Based on his PCA usage,
which one of the following would be an appropriate starting
dosage of a sustained-action morphine (MS Contin)?
15 mg orally twice daily
30 mg orally twice daily
45 mg orally twice daily
60 mg orally twice daily
Answer
• 45 mg orally twice daily
While there is some variability, 10 mg of
parenterally administered morphine is
approximately equivalent to 30 mg orally
(Evidence level I). On average, this patient
has used 29.7 mg of intravenous morphine per
day. This would be approximately equivalent
to 90 mg of oral morphine per day. An
appropriate dosage of sustained-action oral
morphine would be 45 mg twice daily, or 30
mg three times daily. Milligram-to-milligram,
oxycodone is about 50% more powerful than
morphine.
Important concepts for assessing pain in older adults
with cognitive impairment include which of the
following? (Mark all that are true.)
Observing for changes in normal functioning
Asking about pain using synonyms, such as
discomfort, aching, and soreness
Framing questions in the present tense (e.g., "Are
you hurting now?")
Understanding that elderly patients are less sensitive
to pain
Recognizing that persistent pain is likely to affect
physical and psychosocial functioning
Using the 0-10 pain scale, as it works well for nearly
all older adults
Allowing extra time for the patient to assimilate the
questions
Answer
• Observing for changes in normal
functioning
Asking about pain using synonyms, such as
discomfort, aching, and soreness
• Framing questions in the present tense (e.g.,
"Are you hurting now?")
Recognizing that persistent pain is likely to
affect physical and psychosocial
functioning
Allowing extra time for the patient to
assimilate the questions
Persistent pain is common in older adults, particularly among the frail
elderly, in whom cognitive impairment is more common. Age-related
changes in pain perception are probably not clinically significant.
Functional changes, both psychosocial and physical, are common sequelae
to chronic pain and may be the first indicators of pain in cognitively
impaired patients. A substantial portion of older adults (with and without
cognitive impairment) have difficulty using the 0-10 pain scale. Many
other scales have demonstrated their validity in this population (Evidence
level II). Many cognitively impaired older adults deny pain, but may be
able to report distress when synonyms such as "aching" and "soreness" are
used. Focusing on assessment of current symptoms (e.g., asking "Are you
hurting right now?") may also help those with short-term memory deficits.
Validated pain rating systems for the cognitively impaired focus on facial
expressions, posture, vocalizations, appetite, and interactivity (Evidence
level II). Clinicians should choose one tool and use it consistently to
ensure uniformity among health care providers (C level recommendation).
A 40-year-old female with three children has chronic low
back pain and frequent tension headaches. In addition, she
was recently treated for shoulder pain. Her neighbor has
suggested that she look into acupuncture and she asks you if
acupuncture is safe and effective.
Which of the following would be accurate advice? (Mark all
that are true.)
In a randomized study of chronic headache, those treated with
acupuncture in addition to usual therapy had fewer headaches
than controls
The addition of acupuncture to diclofenac (Voltaren) in
patients with shoulder pain improves function more than
diclofenac alone
Studies that looked at more than 60,000 acupuncture
treatments showed no serious adverse events
Answer
• In a randomized study of chronic headache,
those treated with acupuncture in addition
to usual therapy had fewer headaches than
controls
Studies that looked at more than 60,000
acupuncture treatments showed no serious
adverse events
Acupuncture has been practiced for thousands of years and has been used
for hundreds of different ailments. Studies of the method use sham
treatments or minimal treatments as controls. Studies often show
conflicting results or small clinical effects. Acupuncture is quite safe, with
no serious adverse effects reported in two studies including more than
60,000 treatments. Infection is minimized by using disposable needles and
aseptic technique. Serious bleeding is very rare.
In a meta-analysis of chronic back pain studies, acupuncture proved to be
more effective than sham acupuncture or no treatment. For short-term pain
relief in these patients it does not appear to be superior to other active
therapies. It was not particularly effective in acute back pain (Evidence
level I).
As an adjunct to usual therapies, acupuncture has proven effective in
randomized studies of chronic headache and osteoarthritis of the knee
(Evidence level I). It is used as an adjunct in cancer pain management.
One randomized, controlled trial of auricular acupuncture showed a
positive effect in decreasing cancer pain when used with routine
analgesics.
A new patient comes to your office for
evaluation of pain. The patient history
should include which of the following?
(Mark all that are true.)
Identification of possible pain generators
A worker's compensation and litigation
history
A history of the onset and progression of
the pain
A complete medication history
A substance abuse history
Answer
• Identification of possible pain generators
A worker's compensation and litigation
history
A history of the onset and progression of
the pain
A complete medication history
A substance abuse history
In the evaluation of pain, the history may be more valuable than the
physical examination. An important goal of the encounter is to identify the
pain generator when possible, and the history may be the most
illuminating part of the evaluation in this regard (C level
recommendation). Often the specific pain generator cannot be identified.
History taking requires very active listening, with interplay between what
the patient is saying and the physician's interpretation and clarification.
Obtaining a history of the onset and progression of the pain is of great
importance. It can tell the physician whether this is an acute process and if
immediate action is needed (C level recommendation). It also provides
clues as to the amount of additional history that will be needed to sort out
previous treatment successes and failures. A history of legal action related
to pain, for example, is associated with a worse prognosis.
The medication history is a very important part of the initial evaluation.
Rather than just a list of medications the patient is taking, it should include
a discussion of efficacy, tolerability, and economics (C level
recommendation). It might also provide some idea of the patient's attitudes
toward medicines and expectations for efficacy. A history of substance
abuse must be elicited because it has important implications in the
treatment plan and the need for safeguards.
A 30-year-old brick mason presents to your office with midback pain. On examination you note that his rhomboid
muscles are in spasm, and he jumps when you touch three
discrete points in the muscles.
He is concerned that he may be developing fibromyalgia like
his mother. True statements regarding the differentiation
between myofascial pain syndrome and fibromyalgia include
which of the following? (Mark all that are true.)
The tender points of fibromyalgia are different from the
trigger points seen with myofascial pain syndrome
Muscle spasm is most often associated with fibromyalgia
A jump/twitch response is most often associated with
myofascial pain syndrome
The tender points in fibromyalgia patients tend to be
distributed asymmetrically
Myofascial pain tends to be regional
Answer
• The tender points of fibromyalgia are
different from the trigger points seen with
myofascial pain syndrome
A jump/twitch response is most often
associated with myofascial pain syndrome
Myofascial pain tends to be regional
The trigger points seen with myofascial pain syndrome are different from
the tender points seen with fibromyalgia. Trigger points are discrete, focal,
hyperirritable spots located in a taut band of skeletal muscle. Compression
of these points is painful and can produce referred pain, referred
tenderness, motor dysfunction, and autonomic phenomena. Trigger points
may be single or multiple, and are usually asymmetric. Pressing them may
elicit a twitch in the muscle or a jump response from the patient. Trigger
points are associated with regional pain syndromes. Patients with
fibromyalgia exhibit multiple tender points symmetrically distributed along
the axial skeleton, and have constitutional symptoms such as fatigue, sleep
disturbance, and depressed mood.
No single modality stands out as the best for long-term treatment of trigger
points and myofascial pain. However, trigger point injections are widely
accepted and recommended for providing short-term relief (C level
recommendation).
Dry-needle techniques usually result in more soreness the next day than
injection of local anesthetic. The addition of corticosteroids and other
medications to local anesthetics is unnecessary for efficacy and may cause
muscle damage. The technique for trigger point injection is well described
in the reference article.
Common adverse effects of
NSAIDs include which of the
following? (Mark all that are
true.)
Renal toxicity
Gastrointestinal bleeding
Peripheral edema
Increased systolic blood pressure
Answer
• Renal toxicity
Gastrointestinal bleeding
Peripheral edema
Increased systolic blood
pressure
NSAID use is associated with
renal toxicity, gastrointestinal
bleeding and ulcers, peripheral
edema, and increased systolic
blood pressure (median 5 mm
Hg) (Evidence level I).
A 30-year-old female at 38 weeks gestation comes to the hospital with
irregular contractions. Her membranes ruptured spontaneously while she
was at home. She has a history of sciatica, and underwent back surgery 2
years ago. Since then she has been taking opioids for pain control. She is
currently on sustained-release morphine, 15 mg three times daily, and
acetaminophen/oxycodone (Percocet), 5 mg/325 mg every 4-6 hours as
needed for breakthrough pain.
The patient complains of low back pain with contractions but refuses
epidural anesthesia. The resident on call orders nalbuphine (Nubain), 10
mg intravenously every 3 hours as needed for pain. After the first dose,
the pain worsens and the resident approves a repeat dose. The patient
develops severe low back pain, nausea, vomiting, and tremors, and starts
feeling very anxious.
What is the most likely cause of the patient's worsening symptoms?
Underdosing of nalbuphine, as the patient is tolerant to opioids
Increased intensity of labor contractions
Overdosage due to giving a repeat dose of nalbuphine too soon
Decreased opioid effect resulting from fetal absorption
Withdrawal symptoms caused by nalbuphine antagonizing μ receptors
Answer
• Withdrawal symptoms caused by
nalbuphine antagonizing μ receptors
Nalbuphine is an agonist/antagonist opioid
medication. It has an agonist effect on κ receptors.
This is the reason it seems to work better for women,
as they respond better to κ receptor-agonists. Men
respond better to μ receptor-agonists for pain
control. Nalbuphine has an antagonist effect on μ
receptors, which is why it should not be used to treat
pain when patients are on chronic opioid therapy
such as morphine (C level recommendation), which
is a μ receptor-agonist. Nalbuphine's action is similar
to that of naloxone, and it will cause opioid
withdrawal symptoms such as nausea, vomiting,
diarrhea, goose bumps, excessive yawning, tremors,
runny nose, high blood pressure, and anxiety.
Cultural aspects of pain include which of the
following? (Mark all that are true.)
The dominant culture of pain in the United
States honors the stoical person
The meaning and expression of pain is
influenced by the patient's culture
Persons from cultures different from that of
their treating physician often receive
inadequate pain management
Pain behaviors can be predicted reliably by
understanding a patient's culture
To minimize bias, physicians must be aware
of their own pain experiences and culture
Answer
• The dominant culture of pain in the United
States honors the stoical person
The meaning and expression of pain is
influenced by the patient's culture
Persons from cultures different from that of
their treating physician often receive
inadequate pain management
To minimize bias, physicians must be aware
of their own pain experiences and culture
Culture is the framework that directs human behavior in a given situation. The meaning and
expression of pain are influenced by people's cultural background. Pain is not just a
physiologic response to tissue damage, but also includes emotional and behavioral responses
based on individuals' past experiences and perceptions of pain. However, not everyone in
every culture conforms to a set of expected behaviors or beliefs, so trying to categorize a
person into a particular cultural stereotype will lead to inaccuracies. On the other hand,
knowledge of a patient's culture may help to better understand their behavior.
Studies have shown that patients from ethnic minorities and cultures different from the
health care professionals treating them receive inadequate pain management (Evidence level
II). Each of us has the impression that people from distinct cultures are more or less likely to
express their pain experience in a manner that is somewhere between quietly enduring
(stoical) or very expressive. While the physician should attempt to treat the expressive
patient and the stoical patient alike, physicians from a stoical culture are likely to be more
attentive to the patient who is stoical. The culture of pain in mainstream American culture
tends to teach the hurting person to be stoical and the attending person to honor that
stoicism.
For the physician, even more important than understanding the culture of others is
understanding how his or her own upbringing affects attitudes about pain (C level
recommendation). It is important to overcome the belief that one's own reaction to pain is
"normal" and that other reactions are "abnormal." Even subtle cultural and individual
differences between patient and physician, particularly in nonverbal, spoken, and written
language, can affect care.
List three chronic
conditions that may
be effectively
treated by spinal
cord stimulation.
Spinal cord stimulation, also known as dorsal column stimulation, was introduced
in 1967. It has been applied successfully to a number of pain disorders including
angina, tumors, brachial plexus injuries, spinal cord injuries, phantom limb pain,
complex regional pain syndrome/reflex sympathetic dystrophy, ischemic limb
pain, multiple sclerosis, peripheral vascular disease, arachnoiditis, and failed back
surgery syndrome. Success rates are variable. While spinal cord stimulation has
been utilized for a number of pain conditions, there are a limited number of
randomized trials regarding its use (Evidence level I, Cochrane review). More
trials are necessary to confirm that spinal cord stimulation is an effective treatment
for certain types of chronic pain.
Success Rates for Spinal Cord StimulationDiagnosis% Success
Failed back surgery syndrome/low back & leg pain62
Ischemic limb pain77
Complex regional pain syndrome I and II84
Peripheral neuropathy67
Spinal cord injury57
Postherpetic neuralgia82
Stump (phantom limb) pain62
Adapted from Cameron T: Safety and efficacy of spinal cord stimulation for the
treatment of chronic pain: A 20-year literature review. J Neurosurg 2004;100(3
suppl Spine):264.
Spinal cord stimulation is a late resort for chronic intractable pain conditions. The
risks and costs of spinal cord stimulation may outweigh the benefits for many
patients.
A 58-year-old female with metastatic breast cancer
has bone involvement. She is undergoing active
treatment and is still working despite her pain. She
also has nausea from the chemotherapy.
True statements regarding this situation include
which of the following? (Mark all that are true.)
As many as 80% of cancer patients with advanced or
terminal cancer disease have bone metastases
Bone pain is usually sharp, and worsens with resting
Prostaglandin is thought to be involved in cancerrelated bone pain
Irradiation is not effective in the relief of bone pain
from metastases
Relief of bone pain by irradiation, if achieved, is
short-term in effect
Answer
• As many as 80% of cancer patients with
advanced or terminal cancer disease have
bone metastases
Prostaglandin is thought to be involved in
cancer-related bone pain
Metastatic bone pain is troublesome to patients
trying to live and work with their disease. As many
as 84% of advanced or terminal cancer patients have
bone metastases (Evidence level III). The pain is
aching in quality and worse with moving or bearing
weight. Prostaglandins are thought to be involved in
the pain, accounting for the surprisingly good results
with irradiation. About 80% of patients treated with
irradiation will have complete or substantial relief of
their pain (Evidence level I, Cochrane review). Twothirds of those will remain pain free in the irradiated
area for the rest of their lives. The role of
prostaglandins is also the reason for the often
surprising effectiveness of NSAIDs for relief.
A 52-year-old female with type 2 diabetes sees you for a routine followup. Her diabetes is controlled with diet, exercise, and oral
antihyperglycemics. While her diabetes has not always been under good
control, her last hemoglobin A1c was 6.3%. Her cholesterol is under good
control and a recent stress test was negative.
At today’s visit, she describes a painful numbness in her toes bilaterally.
She has been able to continue working but the pain is beginning to
interfere with her sleep. After performing a physical examination, you
decide to treat her symptoms.
True statements regarding this situation include which of the following?
(Mark all that are true.)
A low dose of a tricyclic antidepressant is the preferred initial therapy
SSRIs would be an appropriate first-line therapy if depression were also
present
The efficacy of gabapentin is similar to that of amitriptyline
Opioids, alone or in combination with antidepressants, would be effective
Answer
• A low dose of a tricyclic antidepressant is
the preferred initial therapy
The efficacy of gabapentin is similar to that
of amitriptyline
Opioids, alone or in combination with
antidepressants, would be effective
Meta-analyses consistently show that tricyclic antidepressants (TCAs) are
effective for neuropathic pain (Evidence level I, Cochrane review). They
can be of particular benefit where insomnia, anxiety, or depression is
present. SSRIs are not considered first-line therapy for diabetic
neuropathy because the evidence of their effectiveness is limited
(Evidence level I). Duloxetine and venlafaxine have demonstrated
efficacy in treating neuropathic pain (Evidence level II). An estimated 2.6
patients must be treated with TCAs and 6.7 patients with SSRIs to have
one patient with more than 50% pain relief. Gabapentin has a
demonstrated efficacy in treating neuropathic pain (Evidence level I,
Cochrane review). It is an alternative to TCAs where side effects or
contraindications prevent their use. A small randomized, controlled trial
showed that gabapentin had an efficacy and tolerability similar to that of
amitriptyline (Evidence level II). The efficacy of opioids in the treatment
of neuropathic pain has been consistently demonstrated in randomized,
controlled trials, but they typically require greater caution than other
options (Evidence level I).
A 70-year-old male has significant osteoarthritis in his knees. After failing
conservative measures, he is evaluated for bilateral knee replacement and
expects to undergo surgery in the next few weeks. For pain management,
you have prescribed acetaminophen, 1000 mg orally four times daily, but
the patient is still having significant pain. You wish to improve his pain
control with the use of an acetaminophen/opioid or NSAID/opioid
combination.
True statements regarding these medications include which of the
following? (Mark all that are true.)
Combination medications may improve pain control while limiting the
side effects associated with a higher dose of a single agent
Propoxyphene/acetaminophen (Darvocet N-100) provides pain control
similar to that of acetaminophen alone
Codeine may be ineffective in up to 10% of African-American patients,
due to a cytochrome P450 enzyme deficiency
When the maximum dosage of the acetaminophen or NSAID component
is reached without sufficient pain relief, adding a pure opioid to the
nonopioid is recommended
Unsupervised use of over-the-counter medications along with combination
medication increases the risk of adverse events
Answer
• Combination medications may improve
pain control while limiting the side effects
associated with a higher dose of a single
agent
Propoxyphene/acetaminophen (Darvocet N100) provides pain control similar to that of
acetaminophen alone
Unsupervised use of over-the-counter
medications along with combination
medication increases the risk of adverse
events
The agents in combination medications operate through
different mechanisms. Their use in combination can reduce
the side effects of a higher dosage of a single agent. A metaanalysis of 26 trials involving 2,231 patients compared the
combination of acetaminophen and propoxyphene (Darvocet)
to acetaminophen alone and found that the combination
provided little benefit over acetaminophen (Evidence level I).
To become active, codeine is metabolized to morphine; it may
not be metabolized in up to 10% of Caucasians, due to a
cytochrome P450 deficiency. Combination medications are
limited by their NSAID or acetaminophen content. When the
maximum dosage is reached, switching to non-combination
medications is recommended (C level recommendation).
Without express warnings, patients may use over-the-counter
medications that contain acetaminophen or NSAIDs.
Unsupervised use of these medications increases the risk of
adverse events.
Many acute care and office procedures require
anxiety control and/or pain control. True statements
regarding sedation and pain control for procedures
include which of the following? (Mark all that are
true.)
Reported allergy to lidocaine (Xylocaine) is usually
due to the preservative methylparaben
To decrease injection pain, lidocaine should be
buffered 10:1 with 8.4% sodium bicarbonate
The pain of injection can be decreased by using the
smallest possible needle and injecting slowly
Diphenhydramine 1% (Benadryl) provides
anesthesia comparable to that produced by lidocaine
Answer
• Reported allergy to lidocaine (Xylocaine) is
usually due to the preservative
methylparaben
To decrease injection pain, lidocaine should
be buffered 10:1 with 8.4% sodium
bicarbonate
The pain of injection can be decreased by
using the smallest possible needle and
injecting slowly
Diphenhydramine 1% (Benadryl) provides
anesthesia comparable to that produced by
lidocaine
Guidelines for procedural sedation emphasize that the qualifications of the physician
handling the procedure is the most important criterion when deciding whether to use
procedural sedation. The physician must have an understanding of the medications
administered, must be able to monitor the patient's response to the medications, and must
have the skills necessary to manage all potential complications (C level recommendation).
True allergy to the amine anesthetic lidocaine is rare, and a reaction is most likely due to the
preservative methylparaben (Evidence level III). This can be circumvented by using
preservative-free lidocaine; there is evidence that injection of 1% diphenhydramine
(Benadryl) solution gives anesthesia comparable to lidocaine injection, although it takes a
bit longer to work.
Topical anesthetics can be used to reduce the pain on initial local anesthetic injection, or
topical anesthetics can be used in place of injection. Pain can also be reduced by using the
smallest needle that will work (30 gauge if possible), injecting very slowly, and lightly
pinching the skin around the wound before and during the injection.
Regional nerve blocks are performed by using landmarks to guide the injection of local
anesthetic in the potential spaces around the nerve supplying the area to be numbed. To
avoid systemic toxicity, one must avoid injecting into the arteries and veins in these spaces.
Depending on the size of the nerve and the anesthetic used, it can take 5-20 minutes for a
block to become effective. Epinephrine can be used for local infiltration and field blocks, but
never for nerve blocks. Using a longer-acting anesthetic such as bupivicaine in the nerve
block will make it last longer, but it will take a few minutes longer to work. The blocked
region should be tested before beginning to incise, suture, or cauterize (C level
recommendation).
A 37-year-old male presents to your office with a burn. While
helping his wife with dinner yesterday, he tripped on a dog
bone and put his left hand on a hot stovetop burner. When
you examine the hand, you note that the wound covers the
entire palm, is red and blistered, and blanches under pressure.
He took acetaminophen 2 hours ago, but still rates his pain as
8 on a scale of 10. He seems very uncomfortable.
After cleaning and dressing his wound, which one of the
following would be most appropriate?
No pain medication, because the affected nerve was
destroyed
Cold packs applied to the burn over the next 24-36 hours
Aloe as needed and acetaminophen 4 times daily
A neural blockade to stop the pain
An opioid such as oxycodone/acetaminophen (Percocet)
Answer
• An opioid such as
oxycodone/acetaminophen (Percocet)
Critique for this question:This patient has a partial thickness
burn causing acute somatic pain that is unresponsive to
acetaminophen. An opioid such as oxycodone/acetaminophen
will provide more potent analgesia and is suitable for this
pain profile (C level recommendation). In order to best treat a
burn victim, it is necessary to distinguish the different levels
of burns. This patient has a superficial partial thickness burn.
These burns typically are red and blistered, although blisters
may not appear for 12 hours after the injury. The skin will be
pink and moist under the blister, and the wound surface
blanches with pressure.
Superficial burns can be treated with aloe and acetaminophen
for pain. Neural blockade is generally used for neuropathic
pain management. Although cold packs are initially good to
keep swelling down, they are not suitable for extended pain
management of larger partial thickness burns.
Which one of the following is true regarding the use
of glucosamine and chondroitin sulfate in the
management of chronic osteoarthritis pain?
Treatment with chondroitin sulfate is associated
with a significant decrease in the incidence of joint
swelling, effusion, or both
Glucosamine and chondroitin sulfate in combination
works as fast as celecoxib (Celebrex)
Glucosamine increases the risk of ischemic
cardiovascular events for patients with diabetes
mellitus
The use of glucosamine and chondroitin sulfate is
more beneficial for mild osteoarthritis pain than for
moderate to severe pain
Answer
• Treatment with chondroitin sulfate is
associated with a significant decrease in the
incidence of joint swelling, effusion, or both
The dietary supplements glucosamine and chondroitin sulfate have been advocated, especially in the lay media, as safe
and effective options for the management of symptoms of osteoarthritis. Glucosamine and chondroitin sulfate are the most
widely used dietary supplements for osteoarthritis, with estimated sales in 2004 approaching $730 million.
Several studies have evaluated the efficacy of glucosamine and chondroitin sulfate. Some of these studies have shown that
these supplements are effective, but have been criticized as having flaws such as failure to adhere to the intention-to-treat
principle, enrollment of small numbers of patients, potential bias related to sponsorship of the study by the manufacturers
of the dietary supplements, and inadequate masking of the study agent.
The Glucosamine/Chondroitin Arthritis Intervention Trial (GAIT) was a randomized, double-blind, controlled,
multicenter trial sponsored by the National Institutes of Health. It was designed to rigorously evaluate the efficacy and
safety of glucosamine and chondroitin sulfate, separately and in combination, in the treatment of pain due to osteoarthritis
of the knee. Both placebo and celecoxib were used as control agents.
The GAIT trial showed that glucosamine and chondroitin sulfate, alone or in combination, did not reduce pain effectively
in the overall group of patients with osteoarthritis of the knee. Exploratory analyses suggested that the combination of
glucosamine and chondroitin sulfate may be effective in the subgroup of patients with moderate to severe knee pain
(Evidence level I).
Treatment with chondroitin sulfate was associated with a significant decrease in the incidence of joint swelling, effusion,
or both. There was no increased risk of ischemic cardiovascular events among patients who also received celecoxib, or
among patients with diabetes who received glucosamine.
In the United States, glucosamine and chondroitin sulfate are regulated as dietary supplements and are not held to the
more stringent standards for pharmaceuticals. Substantial variation exists between the content listed on the labels of these
products and the actual product. Because the GAIT trial was conducted under pharmaceutical rather than dietary
supplement regulations, agents identical to the ones used in the study may not be commercially available (Evidence level
III).
In making therapeutic decisions, physicians and patients alike should be aware of data suggesting that celecoxib has a
much faster time to response than glucosamine, chondroitin sulfate, or the two in combination. Continuing research is
needed to establish the potential efficacy and increase our understanding of the biology, pharmacology, and
pharmacokinetics of these agents.
A 68-year-old female completed surgery, radiation, and
chemotherapy for breast cancer 6 years ago. She has recently
developed recurrent breast cancer metastatic to bone. She
does not want further radiation or chemotherapy, and her
oncologist thinks she has less than 6 months to live. She asks
you to continue to be her primary physician as she enters
hospice care.
True statements regarding the treatment of pain in this
situation include which of the following? (Mark all that are
true.)
The management of cancer pain requires specialty
consultation
NSAIDs are contraindicated in cancer patients
Acute or escalating pain requires prompt medical attention
For constant pain with exacerbation, the analgesic regimen
should include a routine baseline dose and breakthrough
dosing
Answer
• Acute or escalating pain requires prompt
medical attention
• For constant pain with exacerbation, the
analgesic regimen should include a routine
baseline dose and breakthrough dosing
• Addiction is rarely an issue in patients with
terminal illness
Cancer pain varies greatly between individuals and during different stages
of the illness. The physician must therefore assess the intensity and quality
and type of pain and choose appropriate interventions. NSAIDs are quite
useful in cancer pain syndromes, particularly bone pain. Escalating pain
that is not promptly addressed will require more drastic and more intense
therapy than pain that is treated promptly (C level recommendation).
Many patients at the end of life have constant pain with exacerbations,
requiring both a routine baseline dose and patient-controlled dosing for
breakthrough pain. A long-acting formulation is typically used for
baseline dosing and a shorter, quicker-acting formulation for acute
exacerbations. Breakthrough doses are usually 10%-30% of the patient's
usual daily dose (C level recommendation).
Addiction behaviors at the end of life are usually seen in active substance
abusers whose addiction predates their terminal illness. Drugs also may be
diverted by family members with active addiction. Terminal patients who
do not have active substance abuse problems will experience tolerance,
but almost never display addictive behaviors.
True statements regarding physical
dependence on opioids include which of the
following? (Mark all that are true.)
Physical dependence develops in most
patients taking opioids on a regular basis for
more than a few weeks
Physical dependence is a marker of addiction
Withdrawal symptoms develop after abrupt
cessation of the opioid
Physical dependence explains why patients
take higher doses than prescribed
Physical dependence explains the symptoms
produced by administration of an opioid
antagonist, such as naloxone (Narcan)
Answer
• Physical dependence develops in most
patients taking opioids on a regular basis for
more than a few weeks
Withdrawal symptoms develop after abrupt
cessation of the opioid
Physical dependence explains the symptoms
produced by administration of an opioid
antagonist, such as naloxone (Narcan)
The American Pain Society, the American Academy for Pain
Management, and the American Society of Addiction Medicine have
jointly created definitions for both physical dependence and addiction.
Physical dependence is defined as "a state of adaptation that is manifested
by a drug class-specific withdrawal syndrome that can be produced by
abrupt cessation, rapid dose reduction, decreasing blood level of the drug,
and/or administration of an antagonist." It is an expected consequence of
chronic opioid use and is distinct from addiction, which is defined as "a
primary, chronic, neurobiologic disease, with genetic, psychosocial, and
environmental factors influencing its development and manifestations. It
is characterized by behaviors that include one or more of the following:
impaired control over drug use, compulsive use, continued use despite
harm, and craving" (C level recommendation).
Withdrawal symptoms may be provoked by sudden cessation of the
opioid, rapid dose reduction, malabsorption or metabolic changes leading
to reduced levels, and/or administration of an antagonist. Physical
dependence does not explain why a patient would take a higher dose than
what was prescribed (Evidence level III).
True statements regarding the use of the
fentanyl transdermal patch (Duragesic)
include which of the following? (Mark all that
are true.)
According to the FDA, it should not be used
for postoperative pain
It should be prescribed at the lowest dose
needed when used on an as-needed basis
Patients may have a sudden rise in their body
level of fentanyl if they become hypothermic
Because fentanyl is absorbed transdermally,
other medications do not affect its plasma
levels
Answer
• According to the FDA, it should not be used
for postoperative pain
The FDA is investigating reports of death and other serious adverse events related
to narcotic overdose in patients using the fentanyl transdermal patch for pain
control. In June 2005 the Duragesic product label was updated to add new safety
information in several areas of labeling, and a "Dear Healthcare Professional"
letter about these changes was issued by the manufacturer.
The directions for use of the fentanyl transdermal patch must be followed exactly
to prevent death or other severe side effects from overdosing. These directions are
provided in the product label and patient package insert.
The fentanyl transdermal patch is a long-acting medication and should not be used
for intermittent pain, short-term pain, or postoperative pain (C level
recommendation). Pain after an operation improves with time and requires a
shorter-acting pain medication for PRN use and for easier titration downward. The
patch delivers a steady level of medication around the clock and cannot be used
for breakthrough pain.
Patients on the fentanyl patch may have a sudden increase in their fentanyl plasma
concentration if they have an increase in their body temperature or are exposed to
heat or use other medicines that increase the elimination half-life of fentanyl.
Ketoconazole is a cytochrome P450 3A4 inhibitor, which is one type of drug that
can have this effect. Compared to other opioids, fentanyl patches are relatively
expensive and are less flexible in terms of dosage titration.
A 5-year-old male has sustained a 3-cm laceration on his
forehead that will require suturing. He is distraught and his
mother is having difficulty getting him to hold still to be
examined. He is afraid of needles and the mother asks if you
can “knock him out” to do the procedure. The boy is allergic
to penicillin.
True statements regarding the relief of pain and anxiety in
this patient include which of the following? (Mark all that are
true.)
A topical mixture of lidocaine, epinephrine, and tetracaine
(LET) can anesthetize the wound without injection
Eutectic mixture of local anesthetics (EMLA) cannot be used
because of the patient's penicillin allergy
Vapo-coolant sprays will effectively control the pain of
intravenous injection for this patient
Combining sedation and analgesia for this patient will require
a dedicated, trained observer and a physician skilled in
pediatric airway management
Answer
• A topical mixture of lidocaine, epinephrine,
and tetracaine (LET) can anesthetize the
wound without injection
Combining sedation and analgesia for this
patient will require a dedicated, trained
observer and a physician skilled in pediatric
airway management
American Academy of Pediatrics guidelines recommend using topical
anesthetics prior to minor procedures (C level recommendation). Eutectic
mixture of local anesthetics (EMLA) and liposomal 4% lidocaine (LMX4)
are effective in numbing intact skin to the pain of venipuncture (including
starting an intravenous line), lumbar puncture, joint aspiration, and
abscess drainage (Evidence level I). EMLA's effect peaks at 1 hour, while
LMX4 takes 30 minutes (Evidence level I). EMLA cannot be used in
patients who have recently taken sulfonamides, but is safe for those with
penicillin allergy. For lacerations, lidocaine/epinephrine/tetracaine (LET)
can be placed in the wound. In about 10 minutes, LET can anesthetize a
skin wound up to 5 cm long with good effect for 20-30 minutes (Evidence
level I).
While vapocoolant sprays decrease the pain of injections, they do not last
long enough to affect the pain of intravenous line insertion (Evidence
level I). For prolonged procedures a combination of analgesia and
sedation may be necessary. Each office or institution must have a policy
regarding this practice. A trained observer must focus on the child's
breathing and circulation (C level recommendation). In addition, a
physician skilled in pediatric airway management must be immediately
available.
True statements regarding the use of methadone for
chronic non-cancer pain include which of the
following? (Mark all that are true.)
Because of methadone's long half-life (>20 hours), it
should be avoided in elderly patients
Methadone's analgesic effect builds over 5-7 days,
producing a duration of analgesia nearly equivalent
to its half-life
Methadone is metabolized principally by the liver
and should not be used in patients with hepatitis C
Methadone, unlike morphine, does not require
adjustment for renal disease
Methadone can be used safely in patients allergic to
morphine
Answer
• Methadone, unlike morphine, does not
require adjustment for renal disease
Methadone can be used safely in patients
allergic to morphine
Methadone is a synthetic opioid with a half-life of about 22
hours. The duration of analgesia is shorter than the half-life,
but typically builds to 8-12 hours with repeated dosing.
Methadone accumulates in the tissues, and serum levels build
gradually over 5-7 days. Dosing adjustments should not be
made more frequently than every 5-7 days in the outpatient
setting (C level recommendation).
While methadone is metabolized by the liver, it is usually
unnecessary to adjust methadone doses for hepatic disease. A
minor portion of methadone is cleared by the kidneys and it is
usually unnecessary to adjust dosing for renal failure.
Because methadone is synthetic, it can be used in patients
with a true allergy to morphine. Methadone should be used
with caution in the elderly patient, and a lower initial dose
may be appropriate (C level recommendation).
A 67-year-old male has recurrent prostate cancer with metastasis to his
lumbar spine. His life expectancy is estimated to be 3 months. He has
already received his maximum dosage of radiation and has elected to
receive only palliative therapies. He is currently using oral morphine and
several adjuvants for pain control. Over the past several weeks, the
amount of morphine required to adequately control his pain is causing
excessive sedation. You are considering a trial of neuraxial
(epidural/intrathecal) administration of pain medications.
True statements regarding this situation include which of the following?
(Mark all that are true.)
Neuraxial administration will maintain analgesia while decreasing
sedation
Side effects such as pruritus, nausea, and urinary retention are rare with
intrathecal administration
When neuraxial medications are used, rescue (breakthrough) doses should
be administered by the same route
Due to the risk of infection, neuraxial techniques are limited to inpatient
use
Answer
• Neuraxial administration will maintain
analgesia while decreasing sedation
For uncontrolled pain or intolerable opioid side effects,
neuraxial (epidural/intrathecal) administration of opioids
should be considered (level C recommendation). Opioids and
local anesthetics can be delivered more efficiently to opioid
receptors, providing analgesia while reducing systemic side
effects. Side effects may still include sedation, nausea,
vomiting, pruritus, constipation, urinary retention, and
respiratory depression. Rescue (breakthrough) medications
may be delivered by any effective route. After an appropriate
trial with a temporary delivery system, neuraxial
administration may be continued using an implanted catheter
and pump, or by using percutaneous catheterization and an
external pump. While the risk of infection exists, the literature
suggests that home parenteral therapy with appropriate
support is effective for analgesia, with no notable risk of
adverse effects.
Goals for treatment of chronic pain
include which of the following?
(Mark all that are true.)
A return to full-time employment
status
Improved physical function
Complete relief of pain
Better quality of life
Improved function in family and
social roles
Empowerment of the patient
Answer
• Improved physical function
Better quality of life
Improved function in family and social
roles
Empowerment of the patient
Empowerment of the patient is the overarching goal of therapy for chronic pain.
The patient is the primary expert on the pain and its response to treatment, as well
as what constitutes realistic goals for therapy. Patient self-report is the most
reliable indication of the presence and intensity of pain (C level recommendation),
and similarly, the patient provides critical input about the level of pain reduction
needed for improved quality of life. Complete relief of chronic pain is seldom a
realistic goal, although reduction in its severity is obviously an important goal.
This should be made clear in the initial partnership with the patient to address
chronic pain.
Improving physical function is another major goal of treatment; explicit steps
toward this end should be negotiated with the patient (C level recommendation).
However, improved physical function will not translate to a full return to work for
many chronic pain patients. Requiring it as a mark of success (or to continue to
"deserve" opioids) would be a mistake.
Depression is a common response to chronic pain, and preexisting depression may
be a risk factor for its development (C level recommendation). All patients with
chronic pain should be assessed for this, along with other psychological
comorbidities. Family issues also commonly arise and should be addressed.
Improved function in family and other social roles is an important goal to include
in the treatment plan for chronic pain (C level recommendation). Input from other
family members (particularly the spouse) may be quite helpful in assessing the
response to treatment of chronic pain.
You are seeing a patient, new to your practice, for failed back surgery
syndrome. He reports he has failed a number of medications and
interventions and requests a prescription for sustained-action oxycodone
(OxyContin). He says that his previous physician had prescribed 20 mg
orally three times a day, and this was providing good relief. He willingly
completes a release which you promptly fax to his previous provider. He
also consents to urine drug testing, part of your office policy for new
patients. In addition to the oxycodone, he reports he is using diazepam
(Valium) as needed for back spasms, and an over-the-counter cold
medication. He denies using any illicit substances.
The results of the immunoassay point–of–care testing tool used by your
office are shown below:
SubstanceCutoff
(ng/mL)ResultMarijuana50NegativeCocaine300PositiveOpioids300Negat
ivePhencyclidine25NegativeAmphetamines1000PositiveBenzodiazepines
200Negative
True statements regarding these results include which of the following?
(Mark all that are true.)
The negative test for opioids rules out oxycodone use
The negative test for benzodiazepines rules out diazepam use
The positive test for cocaine indicates cocaine use within the past 48 hours
The positive test for amphetamines indicates amphetamine use within the
Answer
• The positive test for cocaine indicates
cocaine use within the past 48 hours
Urine drug testing is often used to assist in the diagnosis of drug
abuse/addiction prior to starting opioid therapy and to assist in monitoring
compliance with chronic opioid therapy. Immunoassay testing relies on
the principle of competitive binding and uses antibodies to detect the
presence of a particular drug. Their advantage is the ability to
simultaneously and rapidly test for drugs in urine. When urine drug testing
results are used for legal or employment purposes, the immunoassay
results should be confirmed by mass spectroscopy/gas chromatography.
Immunoassays may only detect certain classes of drugs and may crossreact with other substances, producing inaccurate results. Immunoassays
reliably detect morphine and codeine but do not reliably detect synthetic
and semi-synthetic opioids (e.g., oxycodone, fentanyl, methadone).
The test for cocaine reacts with both cocaine and its primary metabolite,
benzoylecgonine. These tests have low cross-reactivity and are very
specific in predicting cocaine use. The tests for
amphetamine/methamphetamine are highly cross-reactive. They will
detect other sympathomimetic amines such as ephedrine and
pseudoephedrine. The negative result for benzodiazepines may indicate
that the patient has not recently used the medications, or perhaps the test
was not sufficiently sensitive to detect the medication at the concentration
present.
A 20-year-old male skateboarder presents to the emergency department
with severe pain from a broken ankle. You decide to give him parenteral
pain medication but his chart lists an allergy to morphine. When you
question him about this, he says that the last time he had a broken bone he
received morphine and it made him nauseated and he had severe itching
all over.
When determining whether to prescribe an opioid to this patient, which of
the following should be kept in mind? (Mark all that are true.)
Most true allergic reactions to opioids involve codeine, morphine, or
meperidine (Demerol)
Opioid side effects such as nausea, itching, and mild hypotension are often
mistaken for true allergy symptoms
True opioid allergy is a class effect, so allergy to one drug eliminates the
use of all other opioids
Angioedema, severe hypotension, and bronchospasm are signs of true
opioid allergy
Tramadol (Ultram) is a safe substitute for patients with true opioid allergy
Answer
• Most true allergic reactions to opioids
involve codeine, morphine, or meperidine
(Demerol)
Opioid side effects such as nausea, itching,
and mild hypotension are often mistaken for
true allergy symptoms
Angioedema, severe hypotension, and
bronchospasm are signs of true opioid
allergy
Side effects such as nausea, constipation, mild hypotension, and itching are often mistaken for signs of
opioid allergy. True allergy is very rare. Urticaria and itching are often seen with initial doses of opioids.
This is from mast cell release of histamine, and is not IgE-mediated. It can be treated with antihistamines.
When patients have angioedema, severe hypotension, or bronchospasm with opioid use, it must be
considered a true allergy. A drug from a different opioid class can be used, but there are reports of
individuals who are allergic to more than one class of opioids (C level recommendation). Tramadol is also
contraindicated in the presence of true opioid allergy.
Opioid Chemical Classes
Phenanthrenes
Morphine
Codeine
Hydrocodone (Vicodin, Lortab)
Oxydcodone (Percocet, Tylox, OxyContin)
Oxymorphone (Numorphan)
Hydromorphone (Dilaudid)
Nalbuphine (Nubain)
Butorphanol (Stadol)
Levorphanol (Levo-Dromaran)
Pentazocine (Talwin)
Phenylpiperadines
Meperidine (Demerol)
Fentanyl (Duragesic, Actiq, Sumlimaze)
Sufentanil (Sufenta)
Remifentanyl (Ultiva)
Diphenylheptanes
Methadone (Dolophine)
Propoxyphene (Darvon, Darvocet)
A 40-year-old male has chronic pain after three back operations. In
discussing his overall health you discover that he gave up walking a year
ago because of pain, and he has gained 10 lb in the last year. He asks you
if relaxation therapy would help his pain.
You consider the use of exercise, relaxation, or behavioral treatments to
manage the patient's chronic back pain. True statements regarding this
situation include which of the following? (Mark all that are true.)
A Cochrane review found that interdisciplinary biopsychosocial
rehabilitation for >100 hr decreased pain and improved function
Multiple meta-analyses have shown that cognitive therapy modestly
reduces chronic back pain
Passive modalities for back pain, such as TENS, relaxation, and massage,
are recommended in place of active exercise regimens for patients with
limited mobility
Learning to use the relaxation response has been shown to reduce clinic
visits by chronic back pain patients
Answer
• A Cochrane review found that
interdisciplinary biopsychosocial
rehabilitation for >100 hr decreased pain
and improved function
Multiple meta-analyses have shown that
cognitive therapy modestly reduces chronic
back pain
Learning to use the relaxation response has
been shown to reduce clinic visits by
chronic back pain patients
Multiple studies and several guidelines agree that exercise and fitness are
a key component in relieving chronic back pain. No particular type of
exercise demonstrates a clear advantage over any other. Guidelines
recommend fostering self-management skills in patients and setting goals
for functional improvement (Evidence level C). Only very intense
interdisciplinary biopsychosocial interventions made a difference when
reviewed by the Cochrane group (Evidence level I, Cochrane review). A
recent clinical inquiry confirmed the modest effect of cognitive therapies
for improving some chronic pain states, and emphasized the efficacy of
tricyclic antidepressants in reducing chronic back pain (A level
recommendation).
Meditative therapies such as Benson's relaxation response (repetition of a
word, phrase, prayer, or other activity to counteract stress) have been
studied in a variety of contexts. The studies vary in strength of response,
numbers, and quality of controls. It is difficult to make any strong
recommendations for chronic pain, but such therapies are relatively low
cost and harmless (Evidence level III).
True statements regarding the management of
chronic daily headaches include which of the
following? (Mark all that are true.)
A small daily dose of prednisone (5-10 mg)
helps decrease the frequency of headaches
Amytriptyline can reduce headache frequency
by up to 50%
Opioids are effective (50% improvement) in
more than 60% of patients
NSAIDs are associated with a lower risk of
medication overuse headaches compared to
ergotamine
Answer
• Amytriptyline can reduce headache
frequency by up to 50%
NSAIDs are associated with a lower risk of
medication overuse headaches compared to
ergotamine
Chronic daily headache refers to the presence of a headache more than 15 days per month for longer than 3 months. Chronic daily
headache is not a diagnosis but a category that contains many disorders representing primary and secondary headaches. Secondary
causes must be ruled out before the diagnosis of a primary headache disorder is made. Approximately 3% -5% of the population
worldwide and 70%-80% of patients presenting to headache clinics in the United States have daily or near-daily headaches. The
disability associated with this disorder is substantial and includes a diminished quality of life related to physical and mental health, as
well as impaired physical, social, and occupational functioning.
The overuse of medications used for acute headache may lead to medication overuse headache, a syndrome of daily headaches caused
by the very medications used to relieve the pain. The prevalence in the population of chronic daily headache associated with overuse of
these medications was recently estimated to be 1.4% overall, with a higher estimated occurrence among women (2.6%), especially
those over the age of 50 years (5%) (Evidence level II).
Overuse of medications for acute headache is defined as any of the following:
regular overuse of a headache medication for >3 months
use of ergotamine, triptans, opioids, and combination analgesics >10 days/month
use of simple analgesics 15 or more days/month
use of any headache medications 15 or more days/month
NSAIDs and dihydroergotamine mesylate (unlike ergotamine tartrate) are generally associated with a low risk of medication overuse
headache, and are often used to treat breakthrough headaches during the withdrawal period.
Randomized trials of the use of preventive medications in chronic daily headache are scarce. In a single trial involving amitriptyline, the
frequency of headache was reduced by more than 50% in over half of the study participants (Evidence level I).
A recent double-blind, placebo-controlled study evaluated the effect of 100 mg of prednisone for 5 days on the duration of severe
withdrawal headache in 20 patients with presumed medication overuse headache. There was a significant reduction in the number of
hours of severe withdrawal headache in the active-treatment group, which confirmed earlier observations from uncontrolled studies
(Evidence level I).
The use of daily opioid therapy in patients with chronic daily headache is controversial. A recent prospective study with an initial
cohort of 160 patients who were prescribed daily opioid therapy reported the outcomes among 70 patients with medically refractory
chronic daily headache who continued this therapy for at least 3 years. Only 41 of the original 160 patients (26%) had an improvement
of 50% or more on a headache index that took into account the frequency and severity of headaches each week (Evidence level II). Half
of the patients had "problem drug behavior" (defined as "lost" prescriptions, seeking medication from other sources, and most
commonly, dosage violations). Most of these patients (74%) either did not show marked improvement or were dropped from the
program because of problem drug behavior. These data underscore the low efficacy of long-term opioid therapy and the high risk of
misuse in this patient population.
A patient at 32 weeks gestation asks about her
options for the management of labor pain.
Appropriate advice would include which of
the following? (Mark all that are true.)
Epidural anesthesia is associated with
increased rates of cesarean delivery
Epidural anesthesia is associated with
increased instrumentation rates at delivery
Continuous labor support decreases maternal
pain during labor
Warm water baths can decrease labor pain for
short periods of time
Answer
• Epidural anesthesia is associated with
increased instrumentation rates at delivery
Continuous labor support decreases
maternal pain during labor
Warm water baths can decrease labor pain
for short periods of time
Two large meta-analyses have shown little effect of epidural
anesthesia on cesarean delivery rates. Epidural anesthesia
provides better pain relief in labor than opioids, but is
associated with increased instrumentation rates, maternal
fever, and a slightly longer duration of labor (Evidence level
I). A randomized, controlled trial of intrathecal fentanyl
showed superior pain relief and shorter labor duration with no
increase in cesarean delivery rates compared to systemic
hydromorphone (Evidence level I). Reviews of continuous
labor support with doulas show decreased rates of operative
delivery and decreased requests for pain medication
(Evidence level I). Warm water baths appear to decrease pain
but have a short duration of action (Evidence level I). Sterile
saline injections provide short-term relief of back pain but do
not decrease requests for pain medication (Evidence level I).
Despite years of use there remains little clear evidence on the
safety and efficacy of opioids in labor (Evidence level I).
Which one of the following is true
regarding the use of agreements or
contracts for patients using opioids for
chronic pain?
Written agreements reduce the rate of
addiction and abuse
Contracts provide the physician with
legal protection
Contracts are often recommended by
experts in chronic pain management
Contracts have been shown to improve
the patient-physician relationship
Answer
• Contracts are often recommended by
experts in chronic pain management
Contracts or formal agreements are frequently recommended by experts in
chronic pain management, discussed extensively in related literature, and
used by many physicians. While certain goals of contracts could be
supported by virtually all (clarifying plans for use, providing informed
consent, and reducing the risk of addiction), consideration of their use
should include an understanding of their potential negative consequences.
They may damage the patient-physician relationship, erode the patient's
sense of trust and reliance on the physician's beneficence, or provide a
false sense of security for both the patient and the physician about the risk
of addiction (C level recommendation).
Finally, no studies have demonstrated a reduction in the incidence of
addiction or abuse when contracts are used (Evidence level III).
Physicians should be familiar with the legal requirement of their own
states, but the Federation of State Medical Boards model policy on the use
of controlled substances for pain states that the physician should consider
a written agreement for patients at high risk for medication abuse, or for
those with a history of substance abuse (C level recommendation).
A 73-year-old male who is dying of metastatic lung cancer
has been taking sustained-release morphine, 120 mg twice
daily, with occasional use of immediate-release morphine, 60
mg, for breakthrough pain. His pain has been well controlled
by this regimen, but as he nears death he is no longer able to
swallow.
Appropriate strategies for continuing to manage his pain
include which of the following? (Mark all that are true.)
Continue the same dosing regimen of morphine by rectal
suppository
Switch to sublingual morphine, 40 mg every 4 hours
Switch to an equianalgesic dose of methadone for easier
administration
Convert to an equianalgesic dose of fentanyl (Duragesic)
(transdermal and sublingual)
Begin a subcutaneous morphine infusion
Answer
• Continue the same dosing regimen of
morphine by rectal suppository
Switch to sublingual morphine, 40 mg every
4 hours
Convert to an equianalgesic dose of
fentanyl (Duragesic) (transdermal and
sublingual)
Begin a subcutaneous morphine infusion
The preferred route of administration of analgesia for most patients is the oral route. However, when that
route is no longer viable, other routes should be considered.
Limited data, along with expert consensus, suggest that oral opioids, both sustained-release and
immediate-release forms, may be given rectally for equivalent analgesia. (These equivalencies are
generally considered to be true for morphine, hydromorphone and oxycodone. More studies have been
conducted on morphine than the other opioids. Methadone has little data available regarding such
equivalencies.) (C level recommendation.)
This is a simple solution for many patients. However, family members may have both physical and
psychological difficulties in performing rectal administration, and these should be considered.
The pharmacokinetics of sublingual or buccal administration are similar to those of oral administration for
immediate-release preparations. However, sustained-release preparations cannot be administered by this
route.
An opioid infusion, either intravenous or subcutaneous, may also provide a good alternative for selected
patients. In general, the equianalgesic dosage for parenteral administration will be approximately onethird of the oral dose (Evidence level III). Further reductions may be indicated for the starting dose if it is
thought that the oral medications were not being well absorbed.
A transdermal patch, available only for fentanyl at this time, may also be considered, but it is important to
remember that this will not reach a steady state until approximately 18 hours after application. Patients
will require other means of analgesia in the interim and until the correct dosage is established (C level
recommendation). Equianalgesic ranges for fentanyl patches are quite wide. Dying patients may also have
alterations in peripheral circulation and subcutaneous reservoirs that make transdermal patches a less
reliable approach.
True statements regarding the management of fibromyalgia
include which of the following? (Mark all that are true.)
Tricyclic antidepressants or cyclobenzaprine (Flexeril) at
bedtime is an appropriate initial therapy
NSAIDs may be used effectively either as monotherapy or in
combination with other medications
Aerobic exercise two to three times per week may improve
conditioning and fibromylagia symptoms
Sleep and antianxiety agents such as trazodone (Desyrel),
benzodiazepines, and nonbenzodiazepine sedatives are
indicated if sleep disturbance is a prominent symptom
Multidisciplinary approaches that incorporate two or more
strategies help decrease pain and improve function
Answer
• Tricyclic antidepressants or cyclobenzaprine
(Flexeril) at bedtime is an appropriate initial
therapy
Aerobic exercise two to three times per week may
improve conditioning and fibromylagia symptoms
Sleep and antianxiety agents such as trazodone
(Desyrel), benzodiazepines, and
nonbenzodiazepine sedatives are indicated if sleep
disturbance is a prominent symptom
Multidisciplinary approaches that incorporate two
or more strategies help decrease pain and improve
function
The following recommendations regarding the management of fibromyalgia are supported by the strongest evidence
(Strength of evidence A: There is evidence of type I, or consistent findings from multiple studies of types II, III, IV).
Evaluation of the patient with fibromyalgia syndrome (FMS) begins with a complete history and physical examination,
focusing on illnesses that may mimic or complicate FMS, such as hypothyroidism or ankylosing spondylitis, or that can
occur concurrently with FMS, such as tendinitis, systemic lupus erythematosus, rheumatoid arthritis, or osteoarthritis. The
clinician should perform a complete joint examination, manual muscle strength testing, and a neurologic examination.
The clinical diagnosis of FMS depends on the presence of widespread pain, defined as pain in all four body quadrants and
axial pain, for at least 3 consecutive months. The only physical examination criterion for the diagnosis of FMS is the
presence of excess tenderness to manual palpation of at least 11 of 18 muscle-tendon sites.
Multiple strategies, including both pharmacologic and nonpharmacologic therapies, should be used in the management of
FMS. For initial treatment of FMS, a tricyclic antidepressant, in particular 10-30 mg amitriptyline, or cyclobenzaprine can
be given at bedtime to promote sleep. An SSRI such as fluoxetine, alone or in combination with a tricyclic, can be used for
pain relief. NSAIDs should not be used as the primary pain medication. There is no evidence that NSAIDs are effective
when used alone, although NSAIDs (including COX-2 selective agents) and acetaminophen may provide some analgesia
when used with other medications.
Other potentially useful medications include sleep and antianxiety medications such as trazodone, benzodiazepines,
nonbenzodiazepine sedatives, or L-dopa and carbidopa, especially if sleep disturbances such as restless legs syndrome are
prominent.
Patients with FMS should be encouraged to perform moderately intense aerobic exercise (60%-75% of age-adjusted
maximum heart rate) two to three times per week. In individuals who are deconditioned, this rate can be achieved with very
low levels of exercise.
Multidisciplinary approaches incorporating two or more strategies decrease pain and improve function in FMS, especially
in people who have not responded to simpler approaches.
True statements regarding the use of
tramadol (Ultram) include which of
the following? (Mark all that are
true.)
It is a μ opioid receptor agonist
It may lower the seizure threshold
It is an effective agent for
neuropathic pain
It can cause serotonin syndrome
when used with SSRIs
Answer
• It is a μ opioid receptor agonist
It may lower the seizure threshold
It is an effective agent for neuropathic pain
It can cause serotonin syndrome when used
with SSRIs
Tramadol is a weak μ agonist. It also causes both
norepinephrine and serotonin reuptake inhibition and
has the potential to lower the seizure threshold. In a
surveillance study, the risk of seizure was increased
two- to sixfold among users, adjusted for selected
comorbidities and concomitant drug use (Evidence
level III). The risk was highest among those aged 2554 years, those with more than four tramadol
prescriptions, and those with a history of alcohol
abuse, stroke, or head injury. There is also a risk of
serotonin syndrome when tramadol is used with
SSRIs. Tramadol has demonstrated benefits in
neuropathic pain (Evidence level I). Tramadol's side
effects include nausea, constipation, and dizziness.
Its side effect profile is similar to that of codeine.
A 62-year-old female with metastatic cancer of the
colon is expected to die within weeks. She has had
pain in her right chest where a large pulmonary
metastasis has been identified. This pain has been
well controlled with sustained-release morphine, 60
mg twice daily.
What is the most appropriate dose of immediaterelease morphine for breakthrough pain every 1-2
hours on an as-needed basis?
10 mg
30 mg
60 mg
120 mg
Answer
•30 mg
Most experts recommend a starting dose for
breakthrough pain of 10%-25% of the total
daily dose, to be adjusted according to the
patient's response. An oral opioid dose may be
repeated after 1 hour if the response is
inadequate. The patient should be instructed
that recurrent need for more than one dose per
episode of pain is an indication for dosage
adjustment, and the physician should be
contacted. Similarly, a regular need for
breakthrough medication suggests that the
sustained-release dosage should be reassessed.
A 39-year-old male presents to your office with an acute onset of low
back pain. The pain started 2 days ago when he twisted to get out of his
car at the end of a 4-hour drive. He rates the pain as 7 on a 10-point scale,
and describes it as dull and aching. He gets mild relief by lying down and
finds that sitting exacerbates it. The pain is mainly in the left paraspinal
area from L3-L5 with no radiation, and tenderness in that area is the only
abnormal physical finding.
The patient asks your opinion about seeing the chiropractor one of his
friends recommended. Which one of the following would you tell him
about the benefits of manipulation for this problem?
Spinal manipulation is more likely to shorten the duration of his pain than
any other treatments you may prescribe
Spinal manipulation is more likely to reduce the severity of his pain than
any other treatments you may prescribe
Spinal manipulation combined with acupuncture has been found to be the
most effective treatment for his type of pain
Spinal manipulation does not offer any lasting advantages over the other
treatments you usually prescribe
Answer
• Spinal manipulation does not offer any
lasting advantages over the other treatments
you usually prescribe
Acute low back pain is an extremely common
problem with high costs of care. Spinal manipulation
therapy is often recommended, despite contradictory
evidence regarding its effectiveness.
A 2004 Cochrane review concluded that spinal
manipulation was more effective than sham
(placebo) therapy for reducing the severity and
duration of pain, and for improving functional
ability. However, it has not been shown to be more
effective than pain medication, physical therapy,
exercise, back school, or usual primary care
(Evidence level I, Cochrane review).
A 72-year-old male is brought to your office by his daughter
for a routine follow-up visit. He is blind and has type 2
diabetes. He is taken care of at home by the daughter. He
states that he is doing “okay” and that his peripheral
neuropathy and activity level have remained the same since
his last visit 3 months ago. His daughter seems frustrated and
reports that his pain is worse, and that his level of activity has
decreased. When asked to give an example, she says, "Trust
me, his pain is worse." Your examination reveals nothing
remarkable, and the patient’s HbA1c level is 6.3%.
You decide tohave the patient and daughter start a patient
pain and activity log
prescribe an anxiolytic for the daughter to relieve her stress
increase the patient's neuropathic pain medication
Answer
• have the patient and daughter start a patient
pain and activity log
The issues of communication, social relationships, psychological wellbeing, caregiving needs, and spirituality combine to define a complex set
of roles, experiences, and perspectives in the care of loved ones.
Frequently, caregivers report higher levels of pain and immobility than the
patient. On the other hand, patients may be hesitant to admit that they
need more care (Evidence level II). They may understate their pain due to
an increased concern for caregiving needs and future dependency.
Scheduling time with the daughter would help her better understand the
signs of pain, and the factors that influence how patients report their pain
(Evidence level III). A way to obtain a more accurate representation of the
patient's health would be to create a daily patient pain and activity log.
Meeting with the daughter would also facilitate a review of the health
system to ensure that she has the community and psychological support
services that she needs (Evidence level III). This can help prevent anxiety
and depression and reduce unnecessary prescribing of medications.
While there are conflicting reports of pain and activity levels, the
evaluation reveals good control of the patient's glucose levels. In this case,
increasing pain medication would not be indicated. Neither of these two
decisions addresses the patient-caregiver relationship.
A 30-year-old male with AIDS-related neuropathy is
experiencing incomplete pain control. He asks you if
dronabinol (Marinol) or another cannabinoid is
likely to help his pain. Which one of the following
would be accurate advice?
Oral cannabinoids are not associated with CNS
depression or psychotropic effects
Cannabinoids such as dronabinol are no more
effective than codeine
Dronabinol has been shown to be effective as an
adjunct to opioids in treating neuropathic pain
Smoking cannabis has been shown to decrease selfreported pain scores in men with complex regional
pain syndrome
Answer
• Cannabinoids such as dronabinol are no
more effective than codeine
A qualitative systematic review in the British
Medical Journal examined the safety and
efficacy of cannabinoids for treatment of pain.
It showed that cannabinoid doses equivalent
to 5-20 mg of 9-THC were no more effective
than 50-120 mg of codeine, and some were no
better than placebo (evidence level I).
Codeine is the least effective opioid agent. In
this study, cannabinoids were commonly
associated with central nervous system
depression and undesirable psychotropic
effects. Their usefulness for pain control is
therefore quite limited.
True statements regarding the management of
chronic pelvic pain include which of the following?
(Mark all that are true.)
Addressing patients' social issues may be helpful in
resolving symptoms
Chronic pelvic pain patients should be managed by a
gynecologist or pain specialist
When chronic pelvic pain is cyclic, diagnostic
laparoscopy is required before starting hormonal
therapy
When chronic pelvic pain is associated with
gastrointestinal symptoms, a trial of diet
modification and/or antispasmodics is appropriate
A complete evaluation will usually uncover a
specific cause of most chronic pelvic pain
Answer
• Addressing patients' social issues may be
helpful in resolving symptoms
When chronic pelvic pain is associated with
gastrointestinal symptoms, a trial of diet
modification and/or antispasmodics is
appropriate
Addressing psychological and social issues that commonly occur in association
with chronic pelvic pain may be important in resolving symptoms (B level
recommendation). Depression and sleep disorders are common in women with
chronic pain. This may be a consequence rather than a cause of their pain, but
specific treatment may improve the woman's ability to function (Evidence level
III). The multifactorial nature of chronic pelvic pain should be discussed and
explored from the start. The aim should be to develop a partnership between
clinician and patient to plan a management program (A level recommendation).
Many women with chronic pelvic pain can be managed by their primary care
physician. Family physicians might consider referral when the pain has not been
explained to the woman's satisfaction or when pain is inadequately controlled.
Women with cyclical pain should be offered a therapeutic trial using the
combined oral contraceptive pill or a GnRH agonist for a period of 3-6 months
before having diagnostic laparoscopy. The levonorgestrel-releasing intrauterine
system could also be considered (A level recommendation).
Women with associated gastrointestinal symptoms should be offered a trial of
antispasmodics (A level recommendation). They should also try amending their
diet to control symptoms (B level recommendation).
Patients experiencing acute pain due to sickle
cell crisis often obtain poor relief of their
pain. Factors that contribute to this lack of
effective relief include which of the
following?
(Mark all that are true.)
Lack of frequent reassessment of the
effectiveness of treatment
Concerns about addiction
Ethnic minority status
Confusion between addiction, tolerance, and
physical dependence
Lack of effective medicines for this type of
pain
Answer
• Lack of frequent reassessment of the
effectiveness of treatment
Concerns about addiction
Ethnic minority status
Confusion between addiction, tolerance,
and physical dependence
Sickle cell crisis can produce severe pain, and may
require the use of oral, intravenous, or patientcontrolled analgesia. Many of the reasons for poor
relief of pain during sickle cell crisis are the same as
those for poor relief of acute pain in general:
misconceptions and exaggerated concerns about
addiction, low priority given to pain relief, and lack
of appropriate assessment and reassessment. In
addition, ethnic minority status has been shown to
play a role in many health care disparities, including
pain management. Opioids are very effective in
relieving the acute pain of sickle cell crisis (A level
recommendation).
A 64-year-old female has been receiving hospice services for 4 months because of
metastatic breast cancer. She has had right chest wall pain since her right
mastectomy 6 years ago. Since she began hospice care, this has been well
controlled on sustained-release morphine, 30 mg every 12 hours, with rare use of
15 mg every 2 hours as needed.
The hospice nurse calls you on a Monday morning to report that the patient's pain
has become much worse over the previous 24 hours. The patient has been taking
her immediate-release morphine every 2 hours for 18 hours, but still rates the pain
as 8 on a scale of 0-10, and has not slept all night.
Which of the following actions would be appropriate choices for managing this
acute pain?
(Mark all that are true.)
Double the dose of immediate-release morphine and have the nurse call back after
90 minutes to report the effect
Change to a different opioid, beginning with 50% of the equianalgesic dose
Add amitriptyline, 25 mg at bedtime, as an adjuvant analgesic and to help her
sleep better
Have the hospice nurse give 10 mg morphine subcutaneously now, then repeat in
20 minutes if the patient still rates the pain as 5 or greater
Answer
• Double the dose of immediate-release
morphine and have the nurse call back after
90 minutes to report the effect
Have the hospice nurse give 10 mg
morphine subcutaneously now, then repeat
in 20 minutes if the patient still rates the
pain as 5 or greater
This patient is experiencing a pain crisis of rapid onset. Severe pain is a
medical emergency and should be treated as such with aggressive titration
of opioids: a 50%-100% increase in dose is appropriate for initial
treatment, either orally or parenterally. The effect should be assessed
within 90 minutes for oral administration or 15-20 minutes for
subcutaneous administration.
Tolerance to opioids develops gradually, and the patient's history of abrupt
worsening over 24 hours suggests that this does not explain her pain.
Tricyclic antidepressants may be helpful in the management of chronic
pain, but do not typically have a role in the management of a pain crisis.
Opioid rotation is sometimes helpful if pain is refractory to aggressive
titration, but should not be an initial approach to a patient with a pain
crisis.
In addition to rapid treatment of the severe pain, assessment of its etiology
is vital. Appropriate assessment of the sudden onset of severe chest pain in
this hospice patient includes, at a minimum, inquiries about other related
symptoms and physical findings. Further evaluation may be needed for
optimal management.
A 59-year-old female has persistent pain months
after an acute herpes zoster outbreak in a right T-10
distribution. There are no active lesions and only
mild erythema remains where the rash appeared.
Her pain is localized to the back. It is persistent and
severe and interferes with her daily routines.
Which of the following treatment courses would be
good choices?
(Mark all that are true.)
Opioid analgesia
Tricyclic antidepressants
Corticosteroids
Anticonvulsants
Nerve blocks
Answer
• Opioid analgesia
Tricyclic antidepressants
Anticonvulsants
• Nerve blocks
Postherpetic neuralgia can be a severe continuing
problem. Opioids can have a role in the treatment of
acute herpes zoster outbreaks, but in postherpetic
neuralgia should be limited to control of extreme
pain when initiating a treatment plan. Antidepressant
medications, particularly tricyclics, can be very
helpful (Evidence level I). In addition to their role in
pain control, they may be helpful in treating the
depression that can be seen in almost 90% of patients
with postherpetic neuralgia. Corticosteroids are not
useful in postherpetic neuralgia. Anticonvulsants
such as gabapentin are helpful (Evidence level I).
Nerve blocks have been successful in achieving pain
relief, as have transdermal anesthetics.
True statements regarding tricyclic antidepressants include
which of the following?
(Mark all that are true.)
Analgesic efficacy and side effects are both dose related
Because of the anticholinergic side effects, tertiary amines
such as amitriptyline should be avoided in elderly patients
Amitriptyline has demonstrated benefits for acute pain,
neuropathic pain, fibromyalgia, and low back pain
While tricyclic antidepressants can exacerbate existing
cardiac conduction abnormalities, this problem does not arise
with the doses typically used for pain management
The onset of effect for pain relief is similar to that required
for an antidepressant effect
Answer
• Analgesic efficacy and side effects are both
dose related
Because of the anticholinergic side effects,
tertiary amines such as amitriptyline should
be avoided in elderly patients
Amitriptyline has demonstrated benefits for
acute pain, neuropathic pain, fibromyalgia,
and low back pain
Tricyclic antidepressants (TCAs) have been studied in a number of
controlled trials and have demonstrated benefits as analgesics (evidence
level I). Amitriptyline has been studied most thoroughly and is efficacious
in a number of pain states, including acute pain, neuropathic pain,
fibromylagia, and low back pain. Amitriptyline is a tertiary amine and has
significant anticholinergic side effects, including dry mouth, constipation,
urinary retention, sedation, and weight gain. For these reasons,
amitriptyline should be avoided in the elderly (C recommendation). The
secondary amines (desipramine, nortriptyline) have less anticholinergic
activity and may be better tolerated.
Cardiac conduction abnormalities, recent cardiac events, and narrow-angle
glaucoma are contraindications to using tricyclic antidepressants. Both
analgesic efficacy and side effects are dose dependent, with analgesic
efficacy occurring at doses lower than those required to treat depression.
Remarkably, analgesia from TCAs occurs in the absence of depression or
in cases with no antidepressant effect. The onset of analgesia occurs
within 1 week, compared to the 3 weeks required for an antidepressant
effect.
A 45-year-old male is evaluated for a chronic cough and chest
CT shows a suspicious lesion. An open lung biopsy is
planned. The patient has chronic low back pain and takes
methadone, 10 mg orally 3 times a day. He has been on this
stable dose for several years.
You meet with the patient and anesthesia team to plan for
postoperative pain management. True statements regarding
this situation include which of the following?
(Mark all that are true.)
The usual dose of methadone should be continued before and
on the day of surgery, and restarted postoperatively
Alternative analgesia (e.g., epidural) should be used in the
postoperative period while he is NPO
For moderate to severe pain, patient-controlled analgesia and
epidural analgesia have a similar efficacy
The failure rate for epidural analgesia exceeds 15%
Compared to NSAIDs, acetaminophen provides comparable
Answer
• The usual dose of methadone should be
continued before and on the day of surgery,
and restarted postoperatively
Alternative analgesia (e.g., epidural) should
be used in the postoperative period while he
is NPO
The failure rate for epidural analgesia
exceeds 15%
Compared to NSAIDs, acetaminophen
provides comparable analgesia for
postoperative pain
Methadone is a synthetic long-acting opioid used for chronic pain management
and treating opioid addiction. All patients on methadone, whether for methadone
maintenance treatment or pain management, should continue the dose before and
on the day of the surgery to avoid unnecessary fluctuation of the drug level (C
level recommendation). The practice of abrupt discontinuation of methadone
before surgery is unjustifiable. Patients should resume oral methadone as soon as
they can tolerate oral fluids well. During the period of fasting in the postoperative
period, patients should receive alternative analgesia such as intravenous patientcontrolled analgesia or regional analgesia/anesthesia (C level recommendation).
While patient-controlled analgesia (PCA) and epidural analgesia are commonly
used for postoperative pain control, epidural analgesia is generally considered
more effective. There is a lower incidence of moderate to severe pain and severe
pain when an epidural is used (20.9% and 7.8% respectively) compared with PCA
(35.8% and 10.4%) (Evidence level I). Large prospective studies of epidural
analgesia report a 17.4% analgesic failure rate (e.g., catheter dislodgement,
unilateral block, missed segment).
In major surgery, the efficacy of NSAIDs and acetaminophen seems to be
comparable (Evidence level I). Acetaminophen is a viable alternative to NSAIDs,
especially because of the low incidence of adverse effects, and should be the
preferred choice in high-risk patients (C level recommendation).
You have been treating one of your colleagues for chronic
pain over the past several months and you now suspect
addiction problems. True statements regarding impaired
physicians include which of the following?
(Mark all that are true.)
Chemically dependent physicians are more difficult to
identify and treat than regular patients
As many as 10% of physicians abuse alcohol, opioids,
sedatives, or stimulants
When addiction begins to affect coworkers or otherwise
becomes evident in the office or hospital setting, it is a
longstanding problem
Physicians have an ethical obligation to report impaired
colleagues
The physician providing treatment is best suited to monitor
the impaired physician's fitness to work
Answer
• Chemically dependent physicians are more
difficult to identify and treat than regular
patients
As many as 10% of physicians abuse
alcohol, opioids, sedatives, or stimulants
When addiction begins to affect coworkers
or otherwise becomes evident in the office
or hospital setting, it is a longstanding
problem
Physicians have an ethical obligation to
report impaired colleagues
Physicians who are impaired for any reason must refrain from assuming patient responsibilities that they
cannot discharge safely and effectively. Whenever there is doubt, they should seek assistance in caring for
their patients.
Every physician is responsible for protecting patients from an impaired physician and for assisting an
impaired colleague. Fear of being wrong, embarrassment, or possible litigation should not deter or delay
identification of an impaired colleague. The identifying physician may find it helpful to discuss the issue
with the department chair or a senior member of the staff or community.
Impairment may result from use of habit-forming agents (alcohol or other substances) or from psychiatric,
physiologic, or behavioral disorders. Impairment may also be caused by diseases that affect the cognitive
or motor skills necessary to provide adequate care. The presence of these disorders, or the fact that a
physician is being treated for them, does not necessarily imply impairment.
Although the legal responsibility to do so varies among states, there is a clear ethical responsibility to
report a physician who seems to be impaired to an appropriate authority (such as a chief of service, chief
of staff, institutional committee, state medical board, or regulatory agency). Physicians should aid their
impaired colleagues in identifying appropriate sources of help. While undergoing therapy, the impaired
physician is entitled to full confidentiality, as in any other patient-physician relationship. To protect
patients of the impaired physician, someone other than the treating physician must monitor the impaired
physician's fitness to work. Serious conflicts may occur if the treating physician tries to fill both roles.
A study by Hughes and colleagues found that 11.4% of physicians had used benzodiazepines in the
previous year in an unsupervised fashion, and 17.6% had engaged in unsupervised use of opioids
(Evidence level II). Clearly, this results in part from the enhanced access physicians have to these
substances as compared to the general population. At a minimum, these data suggest that a substantial
minority of physicians are using these medications in what would often be considered a clinically
inappropriate fashion. In contrast, this study showed that physicians were less likely than the general
population to have used tobacco and a variety of illicit substances, including marijuana, cocaine, and
heroin, than community controls. Compared with controls, physicians are five times as likely to take
sedatives and minor tranquilizers without medical supervision.
For which of the following
conditions do epidural corticosteroid
injections have moderate to strong
evidence of efficacy?
(Mark all that are true.)
Cervical radiculopathy
Lumbar radiculopathy
Neck pain
Low back pain
Spinal stenosis
Answer
• Cervical radiculopathy
Lumbar radiculopathy
Epidural corticosteroid injections can be administered using
interlaminar, transforaminal, and caudal approaches. While
they are commonly performed by pain specialists, the
evidence of efficacy for these procedures is lacking. Multiple
systematic reviews have provided conflicting opinions. There
is moderate evidence for short-term and long-term relief
using interlaminer epidural injections for cervical
radiculopathy (evidence level 2). The evidence for epidural
injections in managing lumbar radiculopathy was strong for
short-term relief and limited for long-term relief (evidence
level 2). The evidence is inconclusive in the management of
neck pain, low back pain, and lumbar spinal stenosis
(Evidence level C). The evidence is strong for short-term
improvement and moderate for long-term improvement in the
management of nerve root pain.
The Federation of State Medical Boards has
published guidelines for the treatment of pain with
controlled substances. These guidelines recommend
which of the following?
(Mark all that are true.
)Documentation of a complete history and physical
examination
Documentation of the patient's treatment plan,
including ways to measure treatment response
An accurate record of the medications prescribed and
a follow-up plan
Having the patient return at appropriate intervals for
reevaluation
A contract or written agreement between the patient
and the physician
Answer
• Documentation of a complete history and
physical examination
Documentation of the patient's treatment
plan, including ways to measure treatment
response
An accurate record of the medications
prescribed and a follow-up plan
Having the patient return at appropriate
intervals for reevaluation
A contract or written agreement is not required by
the Federation of State Medical Boards (FSMB) but
should be considered if the patient is at high risk for
medication abuse. A contract typically includes
medication dosages, the frequency of visits, and
information about refills of prescriptions. The
guidelines do require a documented treatment plan
and ways to measure treatment response should be
included. Also required are an accurate record of the
medications and a follow-up plan. Central to a good
record, and an FSMB requirement, is documentation
of a complete history and physical examination. The
FSMB and other pain experts believe that the patient
should be seen at appropriate intervals to reevaluate
the treatment plan and assess the patient (C level
recommendation for all recommendations).
True statements regarding chronic pain
include which of the following?
(Mark all that are true.)
It persists after the initial injury has
healed
It can be associated with hyperalgesia
It can be associated with allodynia
It can spread to non-injured areas
It is associated with structural changes in
the central nervous system
Answer
• It persists after the initial injury has healed
It can be associated with hyperalgesia
It can be associated with allodynia
It can spread to non-injured areas
It is associated with structural changes in
the central nervous system
Patients and their physicians are familiar with acute pain or pain caused by injury. Injury leads to inflammation and
changes within the central nervous system. Pain signals are sent to the brain. The brain in turn signals the muscles, causing
a reflex muscle spasm. These changes protect the injured area. The tightening of the muscles forms a natural cast around
the injury, and the negative sensation of pain promotes learning how to avoid similar injury in the future. As tissues heal,
inflammation resolves and the central nervous system sends out fewer signals, resulting in decreased pain and decreased
muscle spasm.
Less is known about the etiology of chronic pain. Chronic pain often occurs in the absence of ongoing illness or after
healing is complete, and often begins with an injury that causes inflammation and central nervous system changes. The
injured area heals, scar tissue is produced, and the inflammation resolves. But, for an unknown reason, the nervous system
continues to send pain signals to somatic muscles, as though a new injury were occurring. The nervous system reacts to the
memory of the original injury and sends signals similar to those sent in response to that injury. These signals become a
disabling message, reminding the patient of the injury (C level recommendation).
Hyperalgesia results when second-order neurons at the level of the dorsal horn become more sensitive to peripheral stimuli.
They demonstrate increased numbers of action potentials and spontaneous discharges in response to painful stimuli. This
increased number of action potentials is experienced as an elevated response to painful stimuli that were previously
perceived as less painful (C level recommendation).
Allodynia is the perception of pain caused by usually nonpainful stimuli, such as touch or vibration. Allodynia results from
a redistribution of central terminals. Mechanoreceptors establish new synapses with dorsal horn cells that normally receive
nociceptive input. After redistribution, mechanoreceptors stimulated by touch or vibration will activate pain pathways in
the same way they are activated by nociceptive neurons in response to pain (C level recommendation).
The spread of pain occurs because of an increase in the size of receptive fields within the dorsal horn. Pain perception then
spreads to involve areas that are not normally innervated by the injured nerve.
Nerve injury may result in multiple changes within the central nervous system that perpetuate the pain experience.
Increased numbers of action potentials cause hypersensitivity to pain. Redistribution of synapses for mechanoreceptors
causes allodynia. Increased receptive field size results in the spread of pain. The use of exercise and psychologic treatment
may be effective in chronic pain because these treatments retrain the nervous system to reestablish more normal neural
connections (C level recommendation).
True statements regarding opioidinduced respiratory depression
include which of the following?
(Mark all that are true.)
It generally occurs in opioid-naive
patients
Pain reduces this effect
It is rare when opioids are
appropriately titrated
It occurs commonly in patients with
COPD
Answer
• It generally occurs in opioid-naive patients
Pain reduces this effect
It is rare when opioids are appropriately
titrated
Respiratory depression generally occurs in opioidnaive patients given excessive initial doses (evidence
level III). It is rare when opioids are appropriately
titrated, even when aggressive rapid titration is
required in the face of severe pain. This side effect of
opioids is reduced by pain, and sudden relief of pain
(such as successful nerve block or a dramatic
response to an adjuvant analgesic) may require rapid
downward titration of the opioid in order to avert
respiratory depression. Patients with underlying
respiratory disease, including COPD, may be more
vulnerable to respiratory depression, but even in
these patients respiratory depression remains rare
when appropriate titration is used.
A 52-year-old male presents to your office with severe,
constant abdominal pain that has kept him home from work
for the last 2 days. When asked about the pain, he states that
he has had similar abdominal pain before, but this time the
pain is much more intense. He is a recovering alcoholic who
has been previously diagnosed with chronic pancreatitis.
True statements regarding pain management in this situation
include which of the following?
(Mark all that are true.)
Opioids cannot be used if a history of alcohol abuse exists
A daily patient pain log is a good way to track chronic pain
Surgery is the definitive treatment for this disease
Pancreatic enzymes coupled with H2-blockers are a good first
treatment
Answer
• A daily patient pain log is a good way to
track chronic pain
Pancreatic enzymes coupled with H2blockers are a good first treatment
Painful chronic pancreatitis is poorly understood, and its management is
controversial. Patients present with a wide variety of pain, ranging from mild to
severe and constant to intermittent. The nature of chronic pancreatic pain only
confounds the response to treatment.
As with any chronic pain situation, it can be difficult to choose the appropriate
pain therapy. This situation is compounded by a history of past or present alcohol
abuse/addiction. Opioids are not ruled out for pain management in alcohol
abusers, but they do need to be selected with an awareness of their addictive
properties. In order to make a more objective decision, it is crucial to assess the
nature, frequency, severity, and activity impact of the pain. A daily patient pain
log is a good way to reach this goal.
Presently, there is no clear evidence that endoscopic or other surgical therapies for
pain in chronic pancreatitis are beneficial. While there have been some promising
results, there have been no controlled trials comparing surgery with either medical
or endoscopic treatment. Further evaluation in clinical trials is needed to define a
good treatment plan.
Before the continuous use of opioids or any surgery is planned, the patient should
be put on a trial of high-dose pancreatic enzymes coupled with H2-blockers (A
level recommendation). This provides a good initial treatment plan that can be
adjusted later.
Which one of the following is true regarding
management of osteoarthritis of the knee?
When prescribing an exercise program,
range-of-motion exercises should be included
Tri-compartmental knee arthritis requires
surgical treatment
Good results have been reported for total knee
arthroplasty in patients over 60 years of age
Arthroscopic lavage should be considered
even for patients without mechanical
symptoms
Answer
• When prescribing an exercise program,
range-of-motion exercises should be
included
Nonpharmacologic osteoarthritis treatment modalities are directed toward weight reduction,
joint protection, and energy conservation. The exercise program should include range-ofmotion (B level recommendation), aerobic, and muscle strengthening exercises. Patients
may also require physical therapy, occupational therapy, assistive devices for ambulation
and activities of daily living, and advice regarding appropriate footwear and orthotics (e.g.,
wedged insoles).
For pharmacologic therapy, the initial drug of choice is acetaminophen, 4 g/day. For patients
taking NSAIDs, gastrointestinal (GI) risk should be assessed, including any history of ulcer
disease and/or GI bleeding. Other risk factors include the use of high-dose, chronic, or
multiple NSAIDs, including aspirin; concomitant use of corticosteroids and/or warfarin (A
level recommendation); and age >60 years. A gastroprotective agent should be prescribed
for patients determined to be at high risk.
Arthroscopic debridement may be indicated for the treatment of patients with degenerative
arthritis with mechanical symptoms, such as locking, catching, or giving way of the joint (B
level recommendation). Neither arthroscopic lavage nor debridement is indicated for
patients without mechanical symptoms (A level recommendation). Results of arthroscopic
debridement in patients with mechanical symptoms are variable, but high success rates are
reported if there is no gross malalignment or instability, some articular cartilage remains,
and symptoms are well localized (B level recommendation).
Patients with bi- or tricompartmental arthritis of the knee should be considered for total knee
arthroplasty only if conservative treatment is unsuccessful (A level recommendation). Good
results have been reported in total knee arthroplasty only in patients under 55 years of age
(A level recommendation).
A 48-year-old male who works on the loading dock at a large distribution
center strained his back 2 days ago while pushing a heavy dolly. When
you see him in your office, he walks haltingly into the examination room
complaining of low back pain. His reflexes and sensation are normal and a
straight leg raising test is negative. His muscle effort is compromised by
pain. He has spasm and tenderness and reduced range of motion in his
lower back.
True statements regarding this situation include which of the following?
(Mark all that are true.)
Close to 90% of patients with low back pain improve within 6 weeks of
onset
One to two days of rest followed by a gradual return to activity gives the
best functional results
All patients over age 45 with low back pain should have lumbar spine
films
Uncontrolled pain is an indication for advanced imaging studies
Any loss of bowel or bladder control requires emergent evaluation
Answer
• Close to 90% of patients with low back pain
improve within 6 weeks of onset
Uncontrolled pain is an indication for
advanced imaging studies
Any loss of bowel or bladder control
requires emergent evaluation
Based on the history and physical examination, low back pain can be classified by duration
and location. Low back pain can be categorized as acute or chronic low back pain, or acute
or chronic sciatica.
Approximately 70% or more of patients with low back pain will improve within 2 weeks,
and 90% will improve within 6 weeks. Back pain that persists for longer than 6 weeks is
classified as chronic. Among patients with chronic pain, 90% have low back pain and the
remainder have sciatica.
Management should emphasize patient education and conservative home self-care that
includes bed rest, early ambulation, postural advice, gentle stretching, use of ice and heat,
nonprescription anti-inflammatory agents and analgesics, and early return to work or
activity. Patients with acute back pain should be advised to stay active, as permitted by the
pain, and patients with chronic pain should be advised that exercise is effective therapy (C
level recommendation).
A small percentage of patients with back pain have a serious underlying disorder, and these
should be taken into consideration during the workup. These disorders include infection,
malignancy, rheumatologic disease, neurologic disease, and problems that can cause pain
referred to the lower back. Patients with signs or symptoms of serious underlying problems
should be seen within 24 hours. Spine radiographs should be performed only if there is a red
flag indicating one of these problems (Evidence level III).
Advanced imaging studies are infrequently indicated. One indication is cauda equina
syndrome, which may be manifested by a sudden loss or change in bowel or bladder control
or function.
• 6. Which of the following is permissible
according to the ethical concept of
therapeutic privilege?
Determining goals of care and treatment
options according to best medical judgment
for
a patient who lacks decision-making
capacity
Disclosing a diagnosis to a patient’s health
care power of attorney Withholding
disclosure of a terminal diagnosis from a
patient at the family’s request Withholding
troubling news temporarily until the patient
is better able to cope with the news
Correct Answer:
Withholding troubling news temporarily until the patient is better able to cope with the news
Explanation:
The therapeutic privilege is the practice of “withholding pertinent medical information from patients
under the belief that the disclosure is medically contraindicated.” It is considered paternalistic and in
direct conflict with patient autonomy. However, it is acceptable and necessary for physicians to assess
how much information a patient is willing or able to accept at a given time, and to sensitively and
respectfully offer full disclosure at an appropriate time. Also, patients may choose not to receive medical
information and/or designate a proxy to receive medical information and make decisions for them.
The ethical concept of therapeutic privilege involves only those patients who have the capacity to make
their own decisions. Disclosing a diagnosis to a patient’s health care power of attorney while the patient
has capacity and wishes to make his/her own decisions is a violation of a patient’s confidentiality, and is
an incorrect use of the durable power of attorney. Withholding a terminal diagnosis at a family’s request is
inappropriate unless the patient waives his/her right to know and defers the medical information and
decisions to the family.
Take Home Message:
•Physicians cannot ethically withhold information from a patient who has capacity to make decisions,
however the physician can use their best judgment as to how much and when to disclose that information
to the patient.
References:
Somerville MA. Therapeutic privilege: variation on the theme of informed consent. Law Med Health Care
1984; 12(1):4-12.
• When bad news is delivered, which of the
following actions is most likely to reassure
the patient and family?
Discussing the next steps to be taken in the
plan of care Offering to call a chaplain or
the family’s personal clergy
member Offering to call the social worker
to review benefits with the patient and
family Sharing personal stories of past
distress as a sign of empathy
Discussing the next steps to be taken in the plan of care
Explanation:
The correct answer is A. After giving bad news and responding to emotions, offering information on treatment during at the
same visit has been shown to have a positive impact on the patient’s view of the conversation. This should be done while
keeping in mind that patients may not remember much after receiving bad news. Aims of treatment may need to be
rediscussed at a later date, as studies have shown that patients often misunderstand the goals of their therapy.
Discussion of the next steps helps everyone to remain grounded on the practicalities of the patient’s care and may provide
hope to the patient and family. Discussing next steps is most likely to reassure the patient and family, as it may help them
know what to expect, lets them know they will not be abandoned by their physician, and prevents them from feeling that
there is nothing more that can be done to help them. Such an approach also serves as a good method to summarize the
discussion.
Offering the assistance of other disciplines can be helpful, but studies have shown that patients do not wish to have other
medical personnel present during the bad news conversation, and reviewing benefits at such an emotional time is
inappropriate. Sharing personal stories is usually not appropriate because it moves the focus of the conversation from the
patient and their concerns, to those of the physician, which is unlikely to be helpful in addressing the patient’s distress.
Take Home Message:
•Discussing “next steps” after giving a patient bad news has a positive impact on the patient’s perspective of the
conversation.
References:
Mackillop WJ, Stewart WE, Ginsberg AD, Stewart SS. Cancer patients’ perception of their disease and its treatment. Br J
Cancer 1988; 58:355-8.
Quirt CF, Mackillop WJ, Ginsberg AD, Sheldon L, Brundage M, Dixon P, et al. Do doctors know when their patients
don’t? A survey of doctor-patient communication in lung cancer. Lung Cancer 1997; 18:1-20.
Schofield PE, Beeney LJ, Thompson JF, Butow PN, Tattersall MHN, Dunn SM. Hearing bad news of a cancer diagnosis:
the Australian melanoma patient’s perspective. Ann Oncol 2001; 12: 365-371.
Yardley SJ, Davis CL, Sheldon F. Receiving a diagnosis of lung cancer: patient’s interpretation, perceptions, and
perspectives. Palliat Med 2001; 15:379-386.
• 1. A 62-year-old man who is dying of
advanced congestive heart failure has
been unconscious for most of the past
24 hours. He has not had much pain
during his illness, but he has now
begun to moan. The family is very
distressed. Which of the following is
the most likely cause of the moaning?
Crescendo pain Depression Spiritual
distress Terminal delirium
Correct Answer:
Terminal delirium
Explanation:
This patient is most likely experiencing terminal delirium. The patient has had significant
mental status change indicating a decline in condition and a hypoactive delirium. The patient
has not had significant pain in the past, and congestive heart failure by itself usually does not
cause pain. In one study of 90 hospice patients with congestive heart failure, the most
prevalent symptom was shortness of breath, but almost half of the patients had delirium. In
the same study, 75% of the patients had no pain, 3% had chest pain, and 20% had other pain.
Depression is unlikely to have such an acute presentation and is unlikely in this situation.
Spiritual distress is possible, but without any prior indication of spiritual pain, it is unlikely
to manifest after a patient has lost consciousness
Take Home Message:
•Delirium is a common symptom at the end of life, and should be considered in the
differential diagnosis of distress.
References:
Travis SS, Conway J, Daly M, Larsen P. Terminal restlessness in the nursing facility:
assessment, palliation, and symptom management. Geriatr Nurs 2001;22:308-12.
Zambrowski CH, Moser DK, Roser LP, Heo S, Chung ML. Patients with heart failure who
die in hospice. Am Heart J 2005; 149:558-64.
• Which of the following is a risk
factor for complicated
bereavement?
Advanced age
• Concomitant illness
• Poverty
• Social isolation
Social isolation
Explanation:
Social isolation is a risk factor for complicated bereavement, because it is a disorder of
attachment and relationships. Other problems with attachment and relationships are also a
risk factor for complicated bereavement as well. Research has shown that survivors whose
relationships were close and dependent with the deceased are a higher risk of complicated
bereavement. Also, those people who had weak parental bonding, people who have suffered
childhood abuse or neglect, those who had difficulties with childhood separation anxiety,
and people who are more adverse to lifestyle changes are at greater risk. Having good
support networks and preparation for the loss has been shown to have lower risk. In addition,
caregivers with a history of substance abuse, mental illness, and poor coping skills are
thought to be more prone to complicated grief.
Advanced age, concomitant illness and poverty, while they are difficult situations, have not
been proven to be independent risk factors for complicated bereavement.
Take Home Message:
•Disorders of attachment and relationships are a risk factor for complicated bereavement.
References:
Ellifritt J, Nelson KA, Walsh D. Complicated bereavement: a national survey of potential
risk factors. Am J Hosp Palliat Care 2003; 20:114-120.
Zhang B, El-Jawari A, Prigerson HG. Update on bereavement research: evidenced-based
guidelines for the diagnosis and treatment of complicated bereavement. J Palliat Med 2006;
9:1188-1203.
• 3. A 54-year-old man with renal cell
carcinoma metastatic to bone has had
involuntary twitching of his muscles for the
past day. His dose of intravenous morphine
has been stable at 80 mg/hr. Three days ago,
his serum calcium level was 12.8 mg/dl, and
he was given pamidronate 90 mg
intravenously. Which of the following is the
most appropriate intervention?
Increase morphine to 150 mg/hr Initiate
clonazepam 0.5 mg three times daily Give
furosemide 40 mg two times daily Initiate
potassium chloride 20 mEq intravenously
Initiate clonazepam 0.5 mg three times daily
Explanation:
Initiating clonazepam would be the most appropriate intervention. The patient has likely
developed acute renal failure after receiving pamidronate, thus reducing the clearance of
neuroexcitatory opioid metabolites. The administration of a benzodiazepine would be
helpful in controlling opioid-induced myoclonus. Other symptoms of opioid neurotoxicity
are delirium, seizures, and hyperalgesia.
Increasing the morphine would be inappropriate as the patient is not having increased pain,
and because this would worsen the myoclonus. The administration of furosemide could
possibly worsen the renal failure. Initiating potassium therapy is potentially dangerous in
patients with renal failure, and therefore is incorrect.
Take Home Message:
•Bisphosphonates can cause renal failure and should be used carefully in patients with
possible renal dysfunction.
•Opioid neurotoxicity may manifest as myoclonus.
References:
Mercandante S. Pathophysiology and treatment of opioid-related myoclonus in cancer
patients. Pain 1998; 74:5-9.
Tanvetyanon T, Stiff PJ. Management of the adverse effects associated with intravenous
bisphosphonates. Ann Oncol 2006; 17:897-907.
• 4. A 64-year-old man with advanced lung
cancer has been intermittently restless,
agitated, and aggressive to his wife for the
past week. Two weeks ago, he decided to
forgo further chemotherapy and
radiotherapy and enrolled in home hospice.
Pain has been well controlled with
transdermal fentanyl 400 mcg/hr and
concentrated morphine solution 100 mg
every two hours as needed. Which of the
following is the most appropriate next step?
Add dexamethasone Add haloperidol Add
lorazepam Discontinue fentanyl and
morphine and initiate hydromorphone
Correct Answer:
Add haloperidol
Explanation:
Adding haloperidol would be the most appropriate next step. The patient is showing signs of a hyperactive
delirium, and therefore should be treated symptomatically with an antipsychotic medication, such as
haloperidol. Antipsychotic medications are considered the first line treatment for delirium.
Adding dexamethasone or lorazepam could possibly worsen the delirium. Benzodiazepines can cause a
“paradoxical response” and steroids can also cause a psychosis. Since there are no other indications of
brain metastasis, such as a focal neurological change, an antipsychotic is a more appropriate first step.
Rotating opioids may be helpful if there were other indications of opioid neurotoxicity, such as
myoclonus. However, the initial management should focus on patient comfort and symptom relief.
Evaluation of the cause of the patient’s delirium should be guided by the patient’s goals of care.
Take Home Message:
•Delirium management should focus initially on control of symptoms with antipsychotics, followed by
work-up and treatment of the etiology as directed by the patient’s goals of care.
References:
Centeno C, Sanz A, Bruera E. Delirium in advanced cancer patients. Palliat Med 2004; 18:184-194.
Gleason OC. Delirium. Am Fam Phys. 2003; 67:1027-34.
Inouye SK. Delirium in older persons. New Engl J Med 2006; 354:1157-65.
Weinrich S, Sarna L. Delirium in the older person with cancer. Cancer 1994; 74: 2079-91.